Photonen sind – das haben die meisten vermutlich schon einmal gehört – die Teilchen, aus denen Licht besteht. Klingt auf den ersten Blick einigermaßen einfach, oder? Fragt man aber acht Physikerinnen, was ein Photon ist, so erhält man etwa zehn Antworten (so geschehen im Jahr 2003 in einer Ausgabe von Optics&Photonics News, Quelle s.u.). Es gibt sogar eine Konferenz-Serie zum Thema: “The Nature of Light: What are photons?”
Also – was ist denn nun ein Photon?

Warnung: Die meisten der Dinge, die ich hier schreibe, stehen so meines Wissens in keinem Buch. Ich bin mir ziemlich sicher, dass alles stimmt, aber ich übernehme wie üblich keine Garantie.

W.E. Lamb, eine berühmte Physikerin*, hat einen Artikel mit dem Titel “Anti-Photon” geschrieben, in dem es heißt:

“It should be apparent from the title of this article that the author does not like the use of the word “photon”, which dates from 1926. In his view, there is no such thing as a photon. Only a comedy of errors and historical accidents led to its popularity among physicists and optical scientists.”

[Aus dem Titel des Artikels sollte offensichtlich sein, dass die Autorin die Verwendung des Wortes “Photon”, das im Jahr 1926 erfunden wurde, nicht mag. In ihrer Sicht gibt es so etwas wie ein Photon nicht. Nur eine Komödie der Irrungen und historische Zufälle haben zu seiner Popularität unter Physikerinnen und optischen Wissenschaftlerinnen geführt.]

*Ja, ich verwende nach wie vor ausschließlich weibliche Formen. Wer sich darüber aufregen will, kann das hier tun; Kommentare bei diesem Artikel hier werde ich schlicht löschen. Und nein, das ist keine Zensur.

Fragen wir erst Mal Wikipedia, für Physik-Fragestellungen meist keine schlechte erste Adresse. Dort lesen wir:

Das Photon (von griechisch φῶς phōs, Genitiv φωτός phōtosLicht‘) ist das Elementarteilchen (Quant) des elektromagnetischen Feldes. Anschaulich gesprochen sind Photonen das, woraus elektromagnetische Strahlung besteht, daher wird gelegentlich auch die Bezeichnung Lichtquant oder Lichtteilchen verwendet.

Das Photon ist also “Das Elementarteilchen des em-Feldes”. Man kann sich laut dieser Erklärung also vorstellen, dass Licht aus Photonen besteht, genauso wie Materie aus Atomen (oder wenn man genauer hinguckt, Elektronen und Quarks) besteht. Klingt erst mal unproblematisch, oder?

Tatsächlich gibt es aber einige Unterschiede zwischen Elektronen (die nehme ich jetzt mal als Beispiel) und Photonen – deswegen werdet ihr vermutlich auch keine Konferenzserie mit dem Titel “What are Electrons?” finden.

Zunächst einmal kann man Elektronen problemlos auf einen sehr kleinen Raum einsperren – ein Elektron kann beispielsweise in einem Quantenpunkt gefangen sein (das ist ein winziges Stück Metall), aus dem es nicht entkommen kann, und dank seiner elektrischen Ladung können wir auch leicht messen, ob es da ist (oder ob es vielleicht sogar zwei oder drei Elektronen sind). Wir können auch die Bahn eines Elektrons verfolgen, beispielsweise in einem Detektor für Elementarteilchen wie einer Blasenkammer:

ELECTRON-POSITRON2-piece

Quelle: CERN, Teachers Web

Hier seht ihr, wie ein einfliegendes (nicht sichtbares) “Photon” ein Elektron-Positron-Paar erzeugt, dessen Weg wir verfolgen können. (Die Wege sind gekrümmt, weil man ein Magnetfeld anlegt; geladene Teilchen werden im Magnetfeld abgelenkt.)

Elektronen können also lokalisiert werden – wir können herausfinden, wo sie sich gerade aufhalten. (Ja, es gibt, die Unschärferelation, die uns da gewisse Grenzen setzt, aber zu einem bestimmten Zeitpunkt kann man ein Elektron beliebig gut an einen Ort festnageln – die Unschärferelation sorgt dann allerdings dafür, dass wir nichts mehr über seine Geschwindigkeit wissen können, so dass im nächsten Moment vermutlich ganz woanders ist. Mehr über die Unschärferelation findet ihr z.B. in der Artikelserie zu Schrödingergleichung – klickt rechts bei den Artikelserien.)

Außerdem sind – wie schon erwähnt – Elektronen elektrisch geladen (deswegen heißen die ja so). Die elektrische Ladung hat zwei Eigenschaften: Zum einen ist sie quantisiert, sie kann also nicht beliebige Werte annehmen. Zum anderen ist sie eine Erhaltungsgröße, Ladungen entstehen also nicht einfach so. Ich kann zwar aus einem ungeladenen “Photon” ein Elektron erzeugen (so wie in dem Bild oben), aber nur, wenn ich gleichzeitig auch ein Teilchen mit einer positiven Ladung erzeuge.

1 / 2 / 3 / 4 / 5 / 6 / 7 / Auf einer Seite lesen

Kommentare (136)

  1. #1 AldiGuru
    6. April 2015

    Bezüglich der Wahrscheinlichkeitsdichte und der Amplitude hab ich das noch nicht ganz verstanden:

    Warum ist bei doppelter Energie auch ein Maximum bei Null?

  2. #2 MartinB
    6. April 2015

    @AldiGuru
    Erst mal kommt das bei der Rechnung einfach so raus. (Die rechnung findest du angedeutet im letzten teil der QFT-Serie, den ich verlinkt habe.) Anschaulich kann man sich das vielleicht so erklären, dass ich bei einer geraden Anzahl von Photonen die beiden destruktiv interferieren lassen kann, aber ob diese Interpretation wirklich wasserdicht ist, weiß ich nicht.

  3. #3 John Sinclair
    7. April 2015

    Beim Doppelspaltexperiment gibt es ja die Variante, dass einzelne Photonen verwendet werden.
    Das Beugungsbild wird dann als Interferenz des Photons mit sich selbst über die zwei möglichen Wege interpretiert. (Bitte korrigieren, falls ich mich irre.)

    Was ich mich aber immer gefragt habe:
    Was ist eigentlich ein einzelnes Photon hierbei und wie stellt man das her?

  4. #4 ulfi
    7. April 2015

    @MartinB Diee Interpretation hört sich sinnig an, wenn man von einer unabhängigen gleichverteilungen der Amplituden ausgeht. Dann muss der Fall das beide schwingungen das selbe Vorzeichen haben genau so groß sein wie das beide negativ sind. Allerdings würde das in deinem zweiten wahrscheinlichkeitsplot bedeuten, dass das mittlere maximum größer ist als die äusseren. (wenn ++ und — genauso wahrscheinlich sind wie +- und -+ dann müssten letztere beiden im mittleren peak zusammengefasst werden)

    passt das so in etwa?

  5. #5 MartinB
    7. April 2015

    @JohnSinclair
    Einzelne Photonen herzustellen, ist letztlich nicht besonders schwierig – das kan man letztlich mit jeder Lichtquelle machen, meist sind die Photonen ja nicht miteinander zu einem kohärenten Zustand verschränkt. Also z.B. mit einer LED.
    Interpretieren tut man das am besten als recht ausgedehntes Wellenpaket, dann ist die Energie einigermaßen scharf definiert.

    @ulfi
    Nein, das mittlere Maximum der Amplitude ist kleiner, soweit ich sehe. (Die Funktionen sind die Hermite-Funktionen, die man auch beim harmonischen oszillator hat.) Weiß nicht, ob man das anschaulich ohne weiteres interpretieren kann.

  6. #6 alex
    7. April 2015

    Ist das Bild zu den kohärenten Zuständen korrekt? Im Text ist die Rede von der Überlagerung von Zuständen unterschiedlicher Photonenzahl, das Bild zeigt aber eine Überlagerung von Zuständen unterschiedlicher Wellenlänge/Frequenz. Das Konzept der kohärenten Zustände kommt doch von einfacheren Systemen, wie etwa dem gewöhnlichen harmonischen Oszillator, wo man es sowieso nur mit einer Frequenz zu tun hat.

    Ich bin mir nicht sicher, ob die Unterschiede zwischen Photonen und Elektronen wirklich in den im Artikel aufgeführten Schwierigkeiten des “Photon”-Begriffs liegen.
    Wenn man nur nicht-wechselwirkende Felder betrachtet (und Wechselwirkungen kommen im Artikel ja nicht vor), sind die einzigen Unterschiede zwischen Elektronen und Photonen die Masse, der Spin und dass Photonen Bosonen und Elektronen Fermionen sind. Masse und Spin kommen im Artikel nicht wirklich vor (bis auf die kurze Anmerkung zur Polarisation), und die einzige Auswirkung der anderen Statistik ist hier, dass es keine Zustände mit mehr als einem Elektron pro Mode gibt. (Das Analogon zu kohärenten Zuständen wäre also deutlich simpler mit lediglich Beiträgen von Vakuum und Ein-Elektronen-Zustand; die e-Funktion ist auf Grassmann-Zahlen linear.)
    D.h. große Teile des Artikels würden auch funktionieren, wenn man überall “Photon” durch “Elektron” ersetzt (und “em-Feld” durch “Diracfeld”, etc.).

    Größere Unterschiede gibt es erst durch die elektrische Ladung der Elektronen, d.h. erst wenn man Wechselwirkungen betrachtet.

  7. #7 TimB
    Bielefeld
    7. April 2015

    Das Photon als Begriff hat mir beim ‘Verstehenwollen’ der Quantenphysik einiges an Kopfzerbrechen und Frust beschert … und mich am weiterkommen gehindert weil ich mir und Google die falschen Fragen gestellt habe.

    Es schmerzt mich immer ein bischen wenn ich irgendwo lese dass Licht aus Photonen besteht oder es sich bei Photonen um Elementarpartikel handelt.

  8. #8 Herr Senf
    7. April 2015

    Was sagt eigentlich der Fotoapparat zu den Photonen?
    Die Dinger hinterlassen doch einen pünktlichen Eindruck,
    wenn sie die Emulsion oder den Chip getroffen haben.
    Vorher waren sie nicht zu “sehen”, hinterher passiert nichts mehr.

  9. #9 Michael
    7. April 2015

    @TimB:
    Was genau meinst Du mit “Elementarpartikel”, und warum sind Photonen keine solchen? Sind Elektronen Elementarpartikel? Und Neutrinos, W-Bosonen, Gluonen und Higgs-Teilchen?

  10. #10 Thorsten
    7. April 2015

    @ MartinB
    Könntest du #3 und #5 doch noch ewas genauer erklären, ich habe mich nämlich auch schon immer gefragt, wie man sicher sein kann, ein einzelnes Photon losgeschickt zu haben, wenn man doch gar nicht so genau weiss, was das eigentlich ist?

  11. #11 MartinB
    7. April 2015

    @alex
    Mist, du hast recht, da hab ich mich vertan.
    Das muss ich korrigieren, Danke.
    “D.h. große Teile des Artikels würden auch funktionieren, wenn man überall “Photon” durch “Elektron” ersetzt”
    Nein, es gibt für Photonen keinen sinnvoll definierten Ortsoperator und auch nicht so etwas wie eine Wellenfunktion, soweit ich weiß (da gibt es diese Wigner-Funktionen, aber die leisten nicht ganz dasselbe.)

    @Herr Senf
    “Was sagt eigentlich der Fotoapparat zu den Photonen?”
    Verstehe die Frage nicht – die Energie wird lokal absorbiert.

    @Thorsten
    “wie man sicher sein kann, ein einzelnes Photon losgeschickt zu haben, wenn man doch gar nicht so genau weiss, was das eigentlich ist?”
    Man kann ja die Energiemenge h nu am Ende messen, das geht ja schon. (Beispielsweise mit nem Photomuliplier oder ner Photodiode, wenn sie extrem empfindlich ist. Auch unser Auge könnte theoretisch einzelne h nu-Quanten wahrnehmen, tut es nicht, aber das ist ne Frage der Rauschunterdrückung.)

  12. #12 alex
    7. April 2015

    @MartinB:
    Naja, in gewissem Sinn sind die klassischen em-Felder die Wellenfunktion des Photons. Was ich meinte war, dass die QFT-Beschreibung von Photonen bzw des elektromagnetischen Feldes (was Du im Artikel beschreibst) nicht so stark von der der Elektronen und des Dirac-Felds abweicht.

  13. #13 MartinB
    7. April 2015

    @alex
    “Naja, in gewissem Sinn sind die klassischen em-Felder die Wellenfunktion des Photons. ”
    Wirklich? Aber daraus bekommt man doch nicht direkt eine Aufenthaltswahrscheinlichkeit, oder?
    “Was ich meinte war, dass die QFT-Beschreibung von Photonen bzw des elektromagnetischen Feldes (was Du im Artikel beschreibst) nicht so stark von der der Elektronen und des Dirac-Felds abweicht.”
    Das stimmt natürlich, aber wie gesagt, mit Elektronen gehen ein paar Dinge (ich kann sie lokalisieren, michins Ruhesystem eines Wellenpakets setzen, die Zahl ist immer definiert…), die mit Photonen so nicht gehen

  14. #14 TimB
    Bielefeld
    7. April 2015

    @Michael:
    Naja Photonen sind die Energiequanten des EM-Feldes. Damit ist ein Photon eine Energieeinheit. Das Wort Partikel enthält das Konzept der massebehafteten Entität mit Trajektorie/Lokalisation und sorgt damit für viel Verwirrung (bspw. beim Durchgang von Licht durch ein transparentes Medium).

  15. #15 alex
    8. April 2015

    @MartinB:
    Wenn man das em-Feld der Maxwell-Gleichungen quantisiert, bekommt man die Quantenfeldtheorie der Photonen, die Du im Artikel beschreibst. Und wenn man das Dirac-Feld (oder in der nichtrelativistischen Näherung das Schrödingerfeld) quantisiert, bekommt man die QFT der Elektronen. Der Begriff “zweite Quantisierung” ist zumindest für Elektronen nicht ganz falsch.

    Dass die Wellenfunktion was mit der Aufentshaltswahrscheinlichkeitsdichte zu tun hat, ist glaube ich ein Spezialfall im nichtrelativistischen Limit. Die Wellenfunktion aus der Klein-Gordon-Gleichung hat diese Eigenschaft nicht und bei Dirac-Spinoren ist es auch schwierig (siehe Klein-Paradoxon). Und Photonen haben halt keinen nichtrelativistischen Limes.

    Sicher gibt es Unterschiede zwischen Photonen und Elektronen, aber ich glaube nicht, dass diese etwas mit den Dingen zu tun haben, die Du im Artikel beschreibst. Photonen und Elektronen sind in genau dem selben Sinn elementare Anregungen eines Quantenfelds.
    Dass Elektronen ein Ruhesystem haben liegt an ihrer Masse; in der Beschreibung als Feld bzw Welle ist das eine Eigenschaft der Dispersionsrelation. Und die kommt im Artikel nur ganz kurz in nicht entscheidener Art vor (wären Frequenz und Wellenlänge nicht antiproportional, würde die Argumentation auch funktionieren).
    Viele andere Unterschiede kommen meiner Meinung nach hauptsächlich davon, dass Elektronen elektrisch geladen sind und deshalb anders mit ihrer Umgebung wechselwirken. Z.B. dass man Elektron und Position im Blasenkammerbild sieht, das Photon aber nicht.
    Es wäre vielleicht interessant, sich die Theorie geladener Photon-ähnlicher Teilchen (also sowas wie masselose W-Bosonen) anzuschauen; insbesondere wüsste ich aus dem Stegreif nicht, wie die Stromdichte für ein solches Feld aussieht.

  16. #16 MartinB
    8. April 2015

    @alex
    “Der Begriff “zweite Quantisierung” ist zumindest für Elektronen nicht ganz falsch.”
    Aber in meinen Augen auch nicht ganz richtig – es wird ja das Feld erst zu einem klassischen feld uminterpretiert und das dann quantisiert. Ist ja nicht so, als würde man doppelt quantisieren.

    “Dass Elektronen ein Ruhesystem haben liegt an ihrer Masse”
    Jupp, aber das hängt doch damit zusammen, dass es einen nicht-relativistischen Grenzfall gibt und damit auch damit, dass man die Bahn eines Elektrons in einer Kammer verfolgen kann, oder siehst du das anders?

    Und ansonsten ist sicher die Tatsache entscheidend, dass es für Photonen eben Überlagerungszustände mit unterschiedlicher Teilchenzahl gibt, die gibt es für Elektronen nicht (und soweit ich sehe auch nicht z.B. für W-Bosonen, weil die auch geladen sind.)

    Mir geht es hier ja auch vor allem darum, zu erklären, auf wie viele unterschiedlcihe weisen man Quantenfelder so basteln kann, dass man das Resultat als “Photon” interpretieren kann – da haben Elektronen eben weniger zu bieten, deswegen ist es da in meinen Augen einfacher.

    Letztlich hast du aber natürlich recht – immerhin gibt es einen schönen Artikel mit dem titel “There are no particles, there are only fields”

  17. #17 ElSaxo
    8. April 2015

    Ich dachte, dass es für Elektronen keine Überlagerungszustände mit unterschiedlicher Teilchenzahl gibt liegt daran, dass es Fermionen sind. Und Photonen eben nicht.

  18. #18 MartinB
    8. April 2015

    @ElSaxo
    Wegen der Ladungserhaltung kann man auch keine Überlagerungszustände von geladenen Bosonen bekommen:
    https://www.spektrum.de/lexikon/physik/superauswahlregeln/14175

  19. #19 alex
    8. April 2015

    @MartinB:
    Ich sehe den Zusammenhang zwischen der Masse der Elektronen und ihrer Verfolgbarkeit in einer Blasenkammer nicht. Sind Elektronen mit Energien >> 511keV in der Blasenkammer nicht sichtbar? Denn im ultrarelativistischen Grenzfall ist ihre Masse ja irrelevant. Die Bahn von Neutrinos ist nicht sichtbar, obwohl sie Masse haben. Liegt das daran, dass sie immer mit quasi Lichtgeschwindigkeit fliegen, oder daran dass sie nur schwach wechselwirken?

    Ja, es gibt eine Superauswahlregel, die Überlagerungen aus Zuständen unterschiedlicher elektrischer Ladung verbietet (wobei ich kürzlich in einem Buch über Dekohärenz die Idee gelesen habe, dass das einfach nur daran liegt, dass die Kohärenz solcher Überlagerungen durch die Wechselwirkung mit der Umgebung extrem schnell verschwindet). Aber es ist möglich Zustände mit einer unterschiedlichen Zahl an Elektronen zu überlagern. Z.B. Vakuum + (1 Elektron & 1 Positron) + (2 Elektronen & 2 Positronen) + … Selbst wenn man Positronen als negative Elektronen zählt, sind Überlagerungen unterschiedlicher Elektronenzahl möglich, z.B. ein freies Neutron nach einer Halbwertszeit.

    Das Bild mit den Elementarteilchen, die man sich intuitiv als kleine harte Kugeln vorstellt, ist (wie so vieles in der Physik) nur ein Bild. In vielen Fällen hilft es, um intuitiv auf das richtige Ergebnis zu kommen, aber es ist keine “fundamentale Wahrheit”.

  20. #20 MartinB
    8. April 2015

    @alex
    Dass man die Teilchen in der Blasenkammer verfolgen kann, liegt natürlich an ihrer ladung. Aber bei der WeWi mit dem Material verlieren sie ja kontinuierlich kinetische Energie (ähnlich bei Phänomenen wie Brems- oder Synchrotron-Strahlung), was bei einem Photon nicht ganz so geht (Ist ein Photon nach einem Compton-Streuprozess noch “dasselbe” Photon? Die Linie im Feynman-Diagramm ist anders als beim Elektron mit Bremsstrahlung jedenfalls nicht durchgezogen.)

    Aber vielleicht ist das auch zu spitzfindig gedacht.

    “Aber es ist möglich Zustände mit einer unterschiedlichen Zahl an Elektronen zu überlagern”
    Das stimmt natürlich, das passiert ja schon, wenn man nen Dirac-Spinor lorentz-boostet. Das sind aber natürlich Elektronen in unterschiedlcihen Zuständen.

    Wie gesagt, vielleicht habe ich den Unterschied stärker dargestellt als das vertretbar ist – ich denke immer noch, dass zum einen die Ladungserhaltung und zum anderen die verschwindende Ruhemasse einen deutlichen Unterschied machen, aber das ist vielleicht auch mehr eine Frage von Interpretation oder Geschmack, über die Physik sind wir uns ja einig, soweit ich sehe.

  21. #21 Artur57
    9. April 2015

    Mal eine Nebensache: die Bahnen des Positrons und des Elektrons, sollten die nicht eigentlich symmetrisch sein? Ladung und Masse sind ja identisch, wohl auch das lokale Magnetfeld. Aber der Unterschied beider Bahnen ist nicht nur auf diesem Foto zu sehen, das ist mir bei allen Aufnahmen bisher aufgefallen. Das Positron macht deutlich mehr Windungen und seine Bahn ist größer. Bei Wikipedia (unter “Paarbildung”) ist das sogar als Zeichnung festgehalten.

    Wobei das Elektron überdies plötzlich verschwindet. Umgekehrt wäre ja einleuchtend, denn das Positron, da Antimaterie, verdampft ja, wenn es mit Pro-Materie in Kontakt kommt.

    Weiß da jemand etwas?

  22. #22 MartinB
    9. April 2015

    @Artur57
    Dass die Bahn kein Kreis ist, sondern zur Spirale wird, liegt ja daran, dass die Teilchen gebremst werden. Ich denke (ist aber nur eine Vermutung), dass Positronen stärker beeinflusst werden, weil sie an negativen Elektronenhüllen stärker abgelenkt werden (eventuell spielt auch das Pauli-Prinzip ne Rolle, dass die Wechselwirkung der Elektronen einschränkt). Ist aber wie gesagt nur eine schnell zusammengebastelte Erklärung.
    Wenn du es genau wissen willst, hilft vermutlich das physicsforum weiter.

  23. #23 Herr Senf
    9. April 2015

    Liegt das nicht nur an den mitbekommenen Impulsen und Richtungen?
    Positronen werden hauptsächlich in die Richtung des erzeugenden Photons emittiert.
    Kleinerer Impuls – kleinere Spirale.

  24. #24 MartinB
    9. April 2015

    @HerSenf
    Stimmt, man braucht ja ne Wewi mit ner Ladung, um die Impulserhaltung sicherzustellen, hatte ich gar nicht drüber nachgedacht. Gibt es ne einfache Erklärung dafür, dass es das Positron sein muss, das mehr Impuls mitbekommt.

  25. #25 Herr Senf
    9. April 2015

    @MartinB,
    wenn Monte Carlo einfach ist 😉
    Steht so unter Paarbildung im wiki, hab’s auch nur gelesen.
    Warum bleibt diese webside ständig hängen?

  26. #26 MartinB
    10. April 2015

    @Herr Senf
    ich hatte gehofft, es gäbeeine einfache Erklärung z.B. weil als Streupartner Kerne involviert sind und die positive Ladung da irgendwas tut (oder weil Elektronen Streupartner sind und dann Pauli irgendwie zuschlägt o.ä.)

    Warum die Website ständig hängt, weiß ich nicht, bei mir zuhause ist sie auch recht langsam, da liegt es am lahmen nameserver meines Internetproviders, der immer ewig braucht, um die ganzen twitter/gravatar/usw-Verbindungen aufzulösen.

  27. #27 Artur57
    10. April 2015

    @Herr Senf:

    was wir hier sehen, ist eine Paarerzeugung eines Positrons und enes Elektrons aus einem Photon. Wikipedia (unter “Paarbildung”) sagt:

    “E(gamma min)beträgt somit näherungsweise 1,022 MeV (Gammastrahlung). Besitzt das Photon eine höhere Energie, so wird diese in die kinetische Energie von Elektron und Positron umgewandelt”

    Ja, doch wohl 1 zu 1. Einen Grund für eine Ungleichverteilug gibt es nicht. Es haben also Positron und Elektron gleiche (wenn auch entgegen gesetzte) Ladung, gleiche Geschwindigkeit und befinden sich im gleichstarken Magnetfeld. Die Kurven sollten also symetrisch sein zur bisherigen Flugbahn des Photons.

    Eine Asymmetrie muss also irgend einen Grund haben.

  28. #28 MartinB
    10. April 2015

    @Artur57
    Wie gesagt, das Photon kann ja nicht spontan zerfallen, es muss Impuls an einen Atomkern oder ein Elektron abgeben. Dadurch wird die Asymmetrie in der Aussendungsrichtugn verursacht werden – wie das passiert, kann ich im Moment nicht anschaulich erklären.

  29. #29 Artur57
    10. April 2015

    @Martin

    Da steht ja etwas sehr Interessantes im Wiki-Artikel: der Zerfall des Photons kann nur in der Nähe eines Atomkerns stattfindfen, denn dieser muss den Impuls aufnehmen.

    Warum Impuls? Nun, aus einem masselosen Photon wird hier plötzlich zwei massebehaftete Teilchen, die sich fortbewegen. Dazu müssen sie einen Impuls bekommen haben, denn es muss stets actio gleich reactio sein. Sonst nämlich verschöbe sich der Schwerpunkt des Universums und das darf nicht sein.

    Wir haben da in der Nähe des Atomkerns natürlich noch andere Störgrößen, die den Effekt beeinflussen. Werde parallel dazu nochmal im Physikerboard fragen. Aber ich denke, es lohnt sich, das im Hinterkopf zu behalten. Irgendwie scheint mir, als ob an dieser Stelle noch etwas Interessantes vergraben wäre.

  30. #30 MartinB
    10. April 2015

    @Artur57
    “der Zerfall des Photons kann nur in der Nähe eines Atomkerns stattfindfen, denn dieser muss den Impuls aufnehmen.”
    Das hatte ich in #24 ja schon gesagt, aber die Frage ist eben, wie die Wewi mit dem Kern (oder nem Elektron) dazu führt, dass die Vorwärtsstreuung für’s ositron wahrscheinlicher wird.

  31. #31 mano
    Möchengladbach
    11. April 2015

    Was hatte Einstein bereits dazu gesagt?
    “50 Jahre Grübelei, was ist ein Photon, haben mich der Antwort keinen Schritt näher gebracht.”

    “Ich wüßte gern, was ein Elektrons ist.”

    Kürzer
    Je präziser, desto unschärfer.

    Das kann man von einer Flasche Bier auch sagen, wenn mehrere in der Realität der Quanten überlagert werden müssen. Damit ist das Welle-Teilchen-Problem aber auch nicht gelöst – erklärt..

  32. #32 Strudel
    13. April 2015

    Danke für den sehr interessante Artikel!

    Eine Sache habe ich nicht verstanden, Du schreibst:
    > Da man elektrische Felder ja messen kann, ist ein Zustand, bei dem sich das Feld
    > zeitlich so ändert wie im Bild oben, niemals ein Zustand mit einer genau definierten
    > Energie, ein solcher Zustand kann also nicht die Gleichung E=hν erfüllen.
    > Wenn ein Photon also ein Quant des em-Feldes mit Energie hν ist, dann ist das,
    > was wir gerade beschrieben haben, jedenfalls kein Photon, denn ein solcher
    > Zustand kann keine genau definierte Energie haben.

    Wieso hat eine klassische stehende em-Welle keine genau definierte und zeitlich konstante Energie? Klar, die Energie fluktuiert zwischen dem E- und B-Feld, aber in der Gesamtheit sollte sie zeitlich konstant sein.

  33. #33 MartinB
    13. April 2015

    @Strudel
    Nein, klassisch geht das, nur in der QM geht es eben nicht – da muss ein Zustand konstanter Energie stationär sein.

  34. #34 Gebhard Greiter
    https://greiterweb.de/spw/Zu_Theoretischer_Physik.htm
    14. April 2015

    Guten Tag, Herr Bäker,

    bisher habe ich (kein Physiker) ein Photon immer verstanden wie unten folgt. Ihrer viel umfangreicheren Beschreibung wegen kommen wir jetzt aber Zweifel. Daher meine Frage: Würden Sie sagen, dass an meiner Vorstellung etwas falsch ist? Und wenn ja, worin genau können Sie mir nicht zustimmen?

    Ich meine:

    Ein Photon ist eine elektromagnetische Welle, d.h.

    — einerseits eine sich kugelförmig (aber um Hindernisse herum gebeugt) ausbreitende Potentialschwankung: eine Schwankung der Potentialhöhe im elektromagnetischen Feld des Universums vergleichbar mit der Schwankung des Wasserpegels in einem ansonsten ruhenden See, die entsteht, wenn man die Wasseroberfläche mal kurz antippt),

    — andererseits aber ist das Photon auch eine Energieportion, deren Größe sich aus der Frequenz jener Welle errechnet.

    Kurz: Meinem Verständnis nach ist jedes Photon wellenförmig schwankendes Wirkpotential, das sich spontan und stets nur komplett irgendwann irgendwo abbauen wird (wobei man jedem Ort im Universum die zeitabhängige Wahrscheinlichkeit zuordnen kann, mit der dieses Ereignis dort eintritt).

    Diese Wahrscheinlichkeit wird orts- und zeitabhängig beschrieben durch das Quadrat einer Lösung der sog. Schrödinger-Gleichung. Die Lösung selbst nennt man die Wellenfunktion des Photons.

    Was da schwingt ist also

    — einerseits ein Kraftpegel (genauer: ein Pegel elektrischer und auch ein Pegel magnetischer, gerichteter Kraft)

    — und andererseits die Wahrscheinlichkeit, mit der an diesem oder jenem Punkt der Raumzeit diese Energieportion mit einem anderen Quant — beispielsweise einem Elektron — verschmelzen wird (was dann den sofortigen Zusammenbruch der das Photon darstellenden Welle überall im Universum bedeutet).

  35. #35 MartinB
    14. April 2015

    @GebhardGreiter
    Ja, das ist die weit verbreitete Vorstellung – sie ist aber eben nicht ganz korrekt.
    Ein einzelnes Photon (ein Zustand mit definierter Photonenzahl 1) ist immer eine Überlagerung aus Zuständen mit mittlerem Wert des elektrischen Feldes von Null.

    Die SGL gilt übrigens nur für Elektronen – und die Frage, in wie weit man so etwas wie eine Wellenfunktion für Photonen definieren kann, ist nicht ganz geklärt. Anschaulich kann man sich das mit Feynman-Diagrammen zusammenreimen, denke ich: Ein Photon kann ich nur detektieren, wenn es mit einer Ladung wechselwirkt. Dabei verschwindet im Diagramm die Photon-Linie. Bei einem Elektron ist das anders. Mathematisch äußert sich das auch darin, dass man für Phtotonen keinen Ortsoperator definieren kann, soweit ich weiß, aber da stoße ich auch an die Grenzen meines QFT-Wissens.

    Das Wort “kraftpegel” und “Wirkpotential” verstehe ich nicht.

    Der letzet Spiegelstrich ist in meinen Augen o.k.

  36. #36 Gebhard Greiter
    14. April 2015

    @MartinB

    Vielen Dank, Herr Bäker. Das hilft mir.

    Hans-Peter Dürr sprach von “Wirks”, wenn er atomare Wirkungen meinte. Da die spontan eintreten (also auch mal ausbleiben, wo sie eintreten könnten), spreche ich von “Wirkpotential” im Sinne von “Energie, die Wirkung verursachen könnte, aber nicht notwendig verursachen muss”.

    Mit dem “Kraftpegel” meine ich die Länge des sich periodisch auf- und abbauenden Vektors elektrischer bzw. magnetischer Kraft, der die elektromagnetische Welle darstellt. Ich könnte mir vorstellen, dass die Höhe der an einem bestimmten Ort jeweils gegebenen Wirkwahrscheinlichkeit mit der Länge dieses Vektors korrespondiert.

    So ganz richtig aber kann das nicht sein, denn sonst müsste ja die Wirkwahrscheinlichkeit dort 1 sein (also dort Wirkung eintreten), wo die Auslenkung des magnetischen oder des elektrischen Teils der Welle maximal wird.

    Was ich auch nicht verstehe: Die Wahrscheinlichkeit, dass das Photon sich mit einem anderen Quant (z.B. einem Elektron) vereinigt, müsste doch auch von jenem anderen Quant anhängen. Letztlich also müsste man wohl die Wellenfunktion des Quantensystems betrachten, welches aus dem Photon und dem Elektron besteht …

    Mich verwirrt auch, dass Sie mir sagen, nur das Elektron habe eine SGL. Muss nicht, wer absolut genau sein will, sagen, dass nur das Universum als Ganzes eine SGL hat (und die des Elektrons nur gute Approximation davon ist in der Region, in der sich der Schwerpunkt der Wolke aus Wirkpotential befindet, die das Elektron darstellt)?

    Und noch eine Frage: Wenn das Elektron eine SGL hat, haben dann vielleicht alle Fermionen eine und alle Bosonen keine? Oder ist es so, dass jedes Elementarteilchen mit Ruhemasse eine SGL hat (was etwas leicht anderes wäre, denn es gibt ja auch Bosonen mit Ruhemasse)?

  37. #37 Gebhard Greiter
    14. April 2015

    Wenn ich den ersten Satz Ihrer Antwort #35 recht verstehe, dann ist ein einzelnes Photon stets ein Paket elektromagnetischer Wellen, die sich — mathematisch — zu Null addieren. Kann man das so ausdrücken?

  38. #38 Gebhard Greiter
    16. April 2015

    @MartinB

    Zu Ihrer Bemerkung (im Post oben), die da lautet:

    Tatsächlich gibt es aber einige Unterschiede zwischen Elektronen (die nehme ich jetzt mal als Beispiel) und Photonen – deswegen werdet ihr vermutlich auch keine Konferenzserie mit dem Titel “What are Electrons? finden.

    2005 hat Volodimir Simulik in einen Buch mit dem Titel » What is the Electron? « 16 Papiere zusammengestellt, deren jedes ein Modell fürs Elektron vorschlägt.

    Die Qualität dieser Papiere — und dieser 16 konkurrierenden Modelle fürs Elektron — kann ich leider nicht beurteilen.

    Siehe https://www.amazon.com/What-Is-Electron-Volodimir-Simulik/dp/0973291125

  39. #39 MartinB
    16. April 2015

    @gebhard
    Wow, das hätte ich nicht gedacht – Leider schweigt sich Amazon ja über den Inhalt aus.
    PS: Wie gut, dass ich “vermutlich” geschrieben habe 😉

  40. #40 Herr Senf
    16. April 2015

    Amazon schweigt, keine Referenzen, dafür worldsci und apeiron, bringt nichts 🙂

  41. #41 MartinB
    16. April 2015

    @HerrSenf
    O.k., dann ist das wohl wenig verlässlich.

  42. #42 Gebhard Greiter
    17. April 2015

    Wie ich immer wieder lese, wird in der Stringtheorie (unter der man heute wohl eine 10-dimensionale Superstringtheorie versteht) jedes Elementarteilchen modelliert als Schwingung eines Strings (oder einer Brane mit bis zu 9 Diemensionen).

    FRAGE also: Muss man sich unter einem Photon dort das Paket aller Schwingungen vorstellen, zu denen der String fähig ist, oder kann ein einziger solcher String auch mehr als ein Photon gleichzeitig realisieren?

  43. #43 MartinB
    17. April 2015

    @gebhard
    Das ist vermutlich (aber ich bin keine String-Expertin) eine etwas falsch gestellte Frage – weil man auch die Strings ja mit den Mitteln der QFT beschreiben muss. Wie man aber einen ausgedehnten n-Photon-Zustand in der Stringtheorie beschreibt, habe ich ehrlich gesagt keine Ahnung.

  44. […] der zugehörigen Quantentheorie wird das em-Feld als ein Photonen-Feld beschrieben. Photonen werden oft auch als “Lichtteilchen” bezeichnet (weil Licht ja eine em-Welle ist). Die […]

  45. #45 Marvin
    28. April 2015

    Hallo!

    Ich habe leider immer wieder ein grundlegendes Problem beim quantenphysikalischen Verständnis von EM-Wellen, welches bis jetzt nie wirklich aufgeklärt werden konnte. Eine ähnliche Frage wurde hier schon gepostet, aber mir ist die Antwort nicht so richtig klar geworden. Bei der Gelegenheit möchte ich gleich auch noch eine weitere Frage stellen, ich hoffe dass das nicht zu lang wird.

    Wenn man eine EM-Welle beispielsweise mit einem Dipol erzeugt, dann breiten sich E- und B-Feld abwechselnd um den Dipol aus und “verschwinden” wieder (um es kurz zu halten drücke ich mich jetzt mal vereinfacht aus, es wieß ja hoffentlich jeder was gemeint ist). Da sich Aufbau und Zusammenbruch der Felder ja nur mit Lichtgeschwindigkeit ausbreiten können, koppeln sich die Felder beim Wechsel also kugelförmig ab und breiten sich ebenfalls kugelförmig als Welle aus. Soweit habe ich hoffentlich noch alles richtig verstanden?

    Wenn man sich jetzt aber das “Photon” als überlagertes Wellenpaket vorstellt, dann ist dieses ja zwar nicht Punktgenau lokalisiert, sondern in einer gewissen Raumregion “verschmiert”, aber dennoch breitet sich dieses Paket ja nicht kugelförmig im ganzen Universum aus.

    Wo also ist die Verbindung zwischen kugelförmiger Welle, und Wellenpaket?

    Ich hatte mir das immer so vorgestellt, dass ein Photon sozusagen einfach die kleinstmögliche Energie ist, die diese EM-Welle abgeben kann. Wie wenn man einen Ball (Analog z. B. zu einem Elektron) im Wasser schwimmen lässt, dem dann ein kleiner Energiebetrag von einer Welle übertragen wird, die den Ball erreicht. Nur mit dem Unterschied, dass die Energieabgabe gequantelt ist, das Feld (also in diesem Modell die Wasseroberfläche) aus überlagerten Zuständen besteht und die Welle nach der Energieabgabe, die sich dann über einen gewissen Radius ausgebreitet hat, überall den Wert 0 besitzt, wenn sie nur die Energie eines einzelnen Photons besaß (andernfalls dehnt sich der Rest der Anregung weiter Wellenförmig aus).

    Des weiteren frage ich mich bezüglich meines oben beschriebenen Verständnis von der Erzeugung einer EM-Welle am Dipol, ob meine (Modell)Vorstellung von der Emission eines Photons bei der Beschleunigung richtig ist. Diesen Vorgang stelle ich mir nämlich ähnlich vor: Wenn ein Elektron beschleunigt wird, so muss diese Geschwindigkeitsveränderung ja auch auf das Elektrische Feld übertragen werden, was aber nur mit Lichtgeschwindigkeit erfolgen kann. Dadurch wird das Feld kurzzeitig vor dem Elektron zusammengedrückt und dahinter gestreckt, und diese Verzerrung erzeugt dann eine Welle, die sich ausbreitet (Das Magnetische Feld lasse ich jetzt mal raus und ich tuhe mal so, als ob ich mich mit dem Elektron bewege, damit sein E-Feld auch ein E-Feld für mich bleibt). Stimmt das so, oder ist die Sache komplizierter?

    Ich weiß, dass meine Fragen wohl aus mangelnder mathematischer Kenntnis über diese Themenbereiche herrührt, aber bevor ich mein Physikstudium beginne (oder mich schon eigenstänndig um das mathematische Wissen kümmere), währe es für mich einfach ganz hilfreich wehnigstens eine relativ realitätsnahe Modellvorstellung der Dinge zu haben… Mal abgesehen davon, dass mein Wissensdurst groß ist, und es einfach seine Zeit braucht sich alles selbst anzueignen xD Daher hoffe ich, dass es nicht schlimm ist, diese Fragen hier zu Posten

  46. #46 MartinB
    28. April 2015

    @Marvin
    “aber dennoch breitet sich dieses Paket ja nicht kugelförmig im ganzen Universum aus.”
    Warum nicht?
    Ist bei Elektronen auch so. Nimm ein Elektron und lokalisiere es an einem Punkt. Dann verschmiert hinterher die WF und zerläuft isotrop in alle Raumrichtungen.

    “Dadurch wird das Feld kurzzeitig vor dem Elektron zusammengedrückt und dahinter gestreckt, und diese Verzerrung erzeugt dann eine Welle, die sich ausbreitet”
    Hm, die Vorstellung finde ich zumindest nicht hilfreich. Ein Photon kann ja auch z.B. ausgesandt werden, wenn ein Elektron von einem Orbital zu einem anderen “springt”.
    Oder meinst du das rein im Rahmen der “klassischen” Elektrodynamik? Dazu gibt es bei den Artikelserien die 6-teilige Maxwell-Gleichungs-Serie, da habe ich ein ähnliches (aber vereinfachtes) Szenario irgendwo diskutiert (ich glaube im letzten Teil). Vielleicht hilft das weiter?

    Ansonsten gern nochmal fragen – Fragen sind hier immer gern gesehen.

  47. #47 Marvin
    29. April 2015

    Hm… ich wurde daraus jetzt immer noch nicht ganz schlau… Also ich versuche das nochmal etwas präziser auszudrücken.

    In dem Artikel haben Sie geschrieben:

    Wenn man sich ein Photon vorstellt, dann denkt man typischerweise an ein Teilchen, das irgendwo losfliegt und dann irgendwann woanders ankommt. […] Wir haben ein elektrisches Feld (das Magnetfeld zeichne ich wie immer nicht mit), das nach außen verschwindet, dann ein Maximum hat und wieder abfällt. Dieses Paket bewegt sich jetzt als ganzes mit Lichtgeschwindigkeit vorwärts.

    Das beschreibt jetzt ja aber ein “Teilchenartiges” Wellenpaket, welches um ein Punkt zentriert ist und sich (von Ortsunschärfen abgesehen) von A nach B in EINE Richtung bewegt.

    Eine von einem Dipol ausgesandte Welle (und ich denke das kann man doch auch so auf solche Übertragen, die von Elektronen emittiert wurden?) breitet sich doch aber konzentrisch von der Quelle weg und das in ALLE richtungen gleichzeitig aus.

    Da sich beide Modelle anscheinend widersprechen, habe ich keine Ahnung, wie ich mir die Ausbreitung der Welle vorzustellen habe.

    Mir ist klar, dass ich durch überlagerung von ebenen Wellen eine art Wellenpaket darstellen kann, aber habe ich mir dann eine kugelförmige EM-Welle auss lauter überlagerten Wellenpaketen vorzustellen (Analog zu einer Art Teilchenstrom) oder ist das Wellepaket einfach nur ein spezieller Zustand in dem sich ein Photon befinden und entsprechend “bewegen” kann?

    Des weiteren frage ich mich, wenn der elektromagnetischen Welle sowohl in der Maxwell`schen Beschreibung als auch in der Quantenfeldtheorie phyikalische Realität zugesprochen wird, gilt das dann auch für eine “Elektronenwelle” in der Quantenfeldtheorie für das Elektron-Feld, oder ist sie wie in der Schrödingergleichung ebenfalls nur eine Wahrscheinlichkeitswelle? Und wie habe ich mir einen Kollaps einer EM-Welle vorzustellen, wenn sie ja nicht nur die Wahrscheinlichkeit darstellt, sondern eine physikalisch reale Störung im EM-Feld?

  48. #48 MartinB
    30. April 2015

    @Marvin
    Ich habe hier den einfachen Fall betrachtet, wo ich das Photon in eine Richtung aussende (zum Beispiel durch eine oszillierende Ladungsverteilung auf einer Platte oder gleich als stehende Welle zwischen zwei Platten, dann gibt es Ausbreitung nur senkrecht zur Platte).
    Bei einem Dipol habe ich tatsächlich eine Welle, die sich kugelförmig nach außen ausbreitet. Wenn ich mir die als “Paket” vorstelle (ich schalte den Dipol kurz an und sende ein pulsartiges Signal aus), dann bildet die Welle eine Kugelschale, die immer größer wird.
    Wenn ich mir jetzt vorstelle, in diesem Paket steckt nur die Energie eines einzigen Photons, dann breitet sich die Welle trotzdem erst mal kugelförmig aus. Ich habe jetzt überall eine Wahrscheinlichkeit, mein Photon zu messen. Anschaulich kannst du dir das auch als Überlagerung aus lauter einfachen Wellenpaketen vorstellen, die in alle Richtungen laufen.
    Wenn ich dann das Photon tatsächlich messe, dann wird eben dieser Zustand sozusagen “realisiert” (die “Wellenfunktion kollabiert”).

    “Und wie habe ich mir einen Kollaps einer EM-Welle vorzustellen, wenn sie ja nicht nur die Wahrscheinlichkeit darstellt, sondern eine physikalisch reale Störung im EM-Feld?”
    Ja, das ist etwas knifflig; ist aber letztlich beim Elektron nicht anders (auch das Elektron ist ja elektrisch geladen und erzeugt deshalb eigentlich ja immer ein E-Feld). Ich denke, man fährt in beiden Fällen am besten, wenn man sich das im Überlagerungsbild vorstellt – es gibt nicht einen eindeutigen Zustand es klassischen Feldes, sondern eben eine Überlagerung dieser Zustände (vielleicht hilft da auch die Qm verstehen-Serie weiter).

  49. #49 Gebhard Greiter
    3. Mai 2015

    @MartinB, @Marvin

    Ein Photon (als Energieportion) in eine Richtung auszusenden scheint mir — streng genommen — gar nicht möglich zu sein.

    Was erzeugt und “ausgesandt” wird ist wohl immer eine Anregung des elektromagnetischen Feldes, die versucht, sich kugelförmig um den Ort herum auszubreiten, von dem sie ausgeht. Erst dadurch, dass sie auf diesem Weg auf Hindernisse trifft, wird sie sich (schwerpunktmäßig) in bestimmte Richtung ausbreiten.

    Zum “Teilchen” wird diese Energieportion erst dann, wenn sie sich mit einer anderen Energieportion (einem Elektron etwa) vereinigt.

    Kurz: Ich würde ein Photon vergleichen mit der aus Gummi bestehenden Haut eines Luftballons, die sich um Hindernisse herum beugt, verformt wird, und schließlich auf ein Hindernis trifft, das sie (wie etwa eine Nadelspitze die Haut des Luftballons) schlagartig um diese Stelle herum zusammenschrumpfen lässt zu etwas, das wir dann ein “Teilchen” nennen (genauer: beobachtbare Wirkung).

    Anders gesagt: Der Teilchencharakter eines Photons besteht, wie ich denke, darin, dass die Lichtwelle — als Wirkung erzeugende Portion von Energie unteilbar, jene Wirkung aber (wenigstens aus Sicht eines Beobachters oder seines Messgeräts) lokalisiert ist.

    VORSICHT ABER: Ich bin kein Physiker. Wollen wir also hören, was MartinB oder seine Kollegen zu meiner Vorstellung sagen.

  50. #50 MartinB
    4. Mai 2015

    @gebhard
    Hmmmmjjjjaaaa – das ist so ein Bild, das irgendwie nicht schlecht ist, aber so ganz passt es auch nicht. Vor allem dieser Teil hier:”schlagartig um diese Stelle herum zusammenschrumpfen lässt” ist in meinen Augen ein bisschen irreführend – bei einer Gummimembran o.ä. müssen sich ja die Teile der Membran alle (mit endlicher Geschwindigkeit) bewegen, während der “Kollaps” ja nach allem was wir wissen instantan verläuft.
    Ist also ein Bild, das ich für eine populärwissenschaftliche Erklärung so halbwegs gelten lassen würde, aber so ganz zufrieden bin ich damit nicht. Ich halte es im Zweifel für besser, einfach zu beschreiben, was passiert (bzw. was wir beobachten/messen können) und das nicht mit Bildern zu unterlegen, die letztlich wieder klassische Physik sind und die Wahrheit nicht einfangen können.

  51. #51 Marvin
    4. Mai 2015

    Danke für die Antworten!

    Naja ich denke, meine Frage wurde damit ganz gut beantwortet. Ich habe ja auch schon recht gute (nicht-Mathematische) Kenntnise und die grundlegendsten Mathematischen über die Quantenphysik, so dass mir diese Konzepte an sich ja schon vertraut sind. Es hatte mich nur immer wieder verwirrt, dass die Welle (nicht im Teilchenmodell) mal als einigermaßen lokalisiertes Wellenpaket und mal um einen großen Radius verteilt und sich immer weiter ausbreitend beschrieben wird.

    Ich hatte neulich nämlich was über die quantenphysikalische Beschreibung von Schallwellen gelesen (hier steht ja auch was zu einer Quantenstimmgabel) und fand die Analogie zwischen Phononen, also dem kleinstmöglichen Energiebetrag einer Schall/Erschütterungswelle und Photonen ziemlich anschaulich. Nachdem ich dann aber diesen Artikel gelesen habe, war ich nach dieser neuen Sicht auf EM-Wellen in analogie zu mech. Wellen wieder Verwirrt, als ich die Beschreibung eines Wellenpaketes gelesen habe, die es bei Schallwellen ja nicht gibt (oder?).

    […] Ich halte es im Zweifel für besser, einfach zu beschreiben, was passiert (bzw. was wir beobachten/messen können) und das nicht mit Bildern zu unterlegen […]

    Aber beschreibt die Quantenfeldtheorie nicht auch was mit der Welle passiert, wenn man nicht misst? Oder war mit “Bilder unterlegen” die Analogie zur klassischen “Gummihaut” gemeint?

    Und zwei Fragen hätte ich noch:

    Die Schrödingergleichung beschreibt ja die Ausbreitung der Welle im Raum… Das selbe tut ja aber auch die Quantenfeldtheorie für zB das Elektron. Sind die in beiden Theorien beschriebenen Wellen die selben (also sind die Wahrscheinlichkeitsverteilung der Schrödingergleichung und die Auslenkungen des Elektron-Feldes das selbe?) Wenn man das zum Beispiel visuell darstellt, dann kann man ja beides auf einer Ebene betrachten und darstellen, welche dann unterschiedliche Auslenkungen haben… Sehen dann also diese visualisierungen beider Theoretischen Beschreibungen gleich aus?

    Und die 2.te Frage: Ich habe mir vor ein paar Monaten einen ziemlichen Wälzer über die theoretische Physik angeschafft, um mich schon einmal auf das Studium vorzubereiten… Allerdings ist mir aufgefallen, obwohl das ziemlich umfangreich ist und eigentlich alle grundlegenden Themenbereiche des Studiums erfasst, behandelt es nicht die Quantenfeldtheorie, obwohl sie ja aus heutiger Sicht die vollständigste und anerkannteste Theorie zur beschreibung der Quantenphysik darstellt.

    Und auch im Studienplan der Uni Hamburg für den Bereich Physik scheint die Quantenmechanik ja zeitlich ziemlich kurz zu kommen daher:

    Behandelt man im Studium die Quantenfeldtheorie, oder muss man sich das Wissen dazu selbst aneignen? Meist wird ja auch in Büchern “nur” die Beschreibung mit Welle-Teilchen-Dualismus über die Schrödingergleichung behandelt und nicht die Beschreibung als Quantenfeld.

  52. #52 Niels
    4. Mai 2015

    @Marvin

    Aber beschreibt die Quantenfeldtheorie nicht auch was mit der Welle passiert, wenn man nicht misst?

    Das ist letztlich eine philosophische Frage, auf die die heutige Physik keine eindeutige Antwort geben kann.

    Sind die in beiden Theorien beschriebenen Wellen die selben (also sind die Wahrscheinlichkeitsverteilung der Schrödingergleichung und die Auslenkungen des Elektron-Feldes das selbe?)

    Nein.

    Behandelt man im Studium die Quantenfeldtheorie, oder muss man sich das Wissen dazu selbst aneignen?

    Das war damals sogar in den Diplomstudiengängen eine rein freiwillige Option. Geprüft wurde das nur, wenn man es absichtlich als Nebenfach bzw. Wahlfach wollte. Und selbst dann geht es nur um “Einführung in die QFT” bzw, “QFT 1”, also um ein Semester lang 5 Wochenstunden.
    Die meisten Diplomphysiker haben dazu nie eine Vorlesung besucht.
    Würde mich wundern, wenn das heute beim Bachelor oder Master anders wäre.

    Ich habe mir vor ein paar Monaten einen ziemlichen Wälzer über die theoretische Physik angeschafft

    Welchen denn, wenn man fragen darf? Zum absoluten Einstieg finde ich die Nolting-Reihe ganz gut.
    Bei mir und praktisch allen anderen war im ersten Semester allerdings die Mathematik das Problem, nicht die Physik. Das allerdings zu Diplomzeiten.

  53. #53 MartinB
    4. Mai 2015

    “als ich die Beschreibung eines Wellenpaketes gelesen habe, die es bei Schallwellen ja nicht gibt (oder?).”
    Klar gibt es die auch bei Schallwellen – jeder kurze Schallpuls ist durch ein WP beschrieben. (Aber quantenmechanisch hat der dann auch keine perfekt scharfe Energie)

    “Oder war mit “Bilder unterlegen” die Analogie zur klassischen “Gummihaut” gemeint?”
    Ja, das war gemeint.

    “also sind die Wahrscheinlichkeitsverteilung der Schrödingergleichung und die Auslenkungen des Elektron-Feldes das selbe”
    Nein, nicht ganz. In der QFT kann man aus dem Elektronfeld z.B. eine lokale Dichte des Elektronsfeldes ableiten, die wiederum lässt sich mit dem WF in Beziehung setzen. Details müsste ich aber nachgucken, die stehen z.B. im Buch von zee, wo die SGL als Grenzfall aus der Diracgleichung abgeleitet wird, wenn ich mich recht entsinne

    “Behandelt man im Studium die Quantenfeldtheorie, oder muss man sich das Wissen dazu selbst aneignen?”
    Als ich seinerzeit in HH Physik studiert hab, waren das Wahrfächer für leute, die sich in theoretischer Physik vertiefen. Ehrlich gesagt habe ich nicht viel davon verstanden, seinerzeit – ich konnte damit rumrechnen, aber was ein QFT-Zustand ist oder wie die ganzen Konzepte zusammenhängen, hat einem niemand erklärt (was ja auch eine Motivation für mich war, es jetzt nochmal richtig zu verstehen und meine QFT-Serie zu schreiben). Generell kamen anschauliche Erklärungen im Studium sehr sehr kurz. (Über grottig schlechte Physik-Didaktik wollte ich auch immer mal was schreiben…)

  54. #54 Marvin
    5. Mai 2015

    @Niels: https://www.amazon.de/gp/aw/d/364254617X/ref=mp_s_a_1_1?qid=1430821065&sr=8-1&pi=AC_SX110_SY165_QL70&keywords=theoretische+physik+springer

    @MartinB: Also ich habe mir gestern, um diesen Zusammenhang zwischen klassischen Feldern, Wahrscheinlichkeitswellen und QFT zu verstehn, ein paar Simmulationsapplets aus dem Internet geladen, um mir so eine Welle, die ja meistens nur 2-Dimensional als Graph gezeichnet ist 3-Dimensional visualisieren zu können. Dabei kann man einzelne Photonen und Elektronen oder mehrere in verschiedenen Situationen darstellen lassen und sieht sehr schön die Ausbreitung der Wellen. Zusätzlich kann man zwischen verschiedenen Darstellungen wählen: Wahrscheinlichkeitswelle so wie es die Schrödingergleichung darstellt (oder beim Photon einfach die Wahrscheinlichkeitsverteilung), EM-Welle und man kann die direkte Wellenfunktion betrachten (also ohne das Amplitudenquadrat zu nehmen). Leider habe ich keine Simmulation für QFT-Felder gefunden.

    Aber zumindest kann ich bei der Wellenfunktion bzw der Wahrscheinlichkeitsverteilung sagen, dass diese Welle sich ebenso ausbreitet wie die EM-Welle. Also würde man die Wellen im Raum umranden und nur die Form, Position usw betrachten (wie zB die schon genannte Gummihaut oder ein Nebel), dann breiten sich beide Wellen gleich aus und verhalten sich gleich. Natürlich gibt es Mathematische und Interpretationstechnische Unterschiede, aber darum geht es mir ja (noch) nicht. Und für mich ist es eigentlich auch ganz plausibel, denn da wo das (nach den Maxwellschen Gleichungen beschriebene) EM-Feld keine Störung aufweist, wird man doch bei einer Messung auch kein Photon messen oder?

    Wie gesagt, es ging mir nur darum, ob beide die gleiche Welle beschreiben, hinsichtlich ihrer Ausbreitung und ihres Verhaltens (Beugung, Interferenz usw).

    Und hinsichtlich dieser Betrachtungsweise würde mich dann eben noch der Unterschied zu den QFT-Feldern interessieren ( ebenfalls nur hinsichtlich der Ausbreitung und des Verhaltens einer Störung). Denn soweit wie ich das Verstanden habe (in ihrer QFT-Reihe), beschreibt doch zB die QED die selbe EM-Welle wie die Maxwellsche Beschreibungen, nur mit den Quantenmechanischen Modifikationen der Superposition und Unschärferelation oder?

    Aber ich werde mir auch nochmal alle Reihen zu diesen Themen hier genau durchlesen. Würde mich aber trotzdem noch über Antworten freuen, damit ich einfach einen besseren Bezug zwischen den 3 genannten Wellenbeschreibungen und der Realität herstellen kann.

  55. #55 MartinB
    6. Mai 2015

    @Marvin
    “dass diese Welle sich ebenso ausbreitet wie die EM-Welle.”
    Ja, das ist schon richtig – die beiden Wellen müssen ja an denselben Orten von Null verschieden sein. (Naja, auch nicht ganz zwingend, wenn man ans Vektopotential und so Dinge wie den Aharonov-Bohm-Effekt denkt…)

    “beschreibt doch zB die QED die selbe EM-Welle wie die Maxwellsche Beschreibungen, nur mit den Quantenmechanischen Modifikationen der Superposition und Unschärferelation oder?”
    Ja – oder jein. Die Unschärferelation ist ja mehr ein ergebnis der Rechnung, nicht etwas, was man am Anfang reinsteckt. Aber ich denke, als prinzipielle Vorstellung und Anschauung ist das, was du (eigentlich duzen sich hier auf dem Blog (fast) alle) schreibst, völlig in Ordnung.

  56. #56 Gebhard Greiter
    6. Mai 2015

    Ist die Vorstellung, wirklich jedes elementare Teilchen sei eine Welle oder ein Paket von Wellen falsch?

    Wenn richtig: Müsste nicht jede der Wellen, die Teil eines dieser Pakete sind, eine der 4 Grundkräfte der Physik übertragen?

    Konkreter: Elektronen bewegen sich – anders als Photonen – ja offenbar nicht mit Lichtgeschwindigkeit durch den Raum. Erklärbar wäre das aber sehr wohl, wenn jedes Elektron ein ganzes Paket von Wellen wäre: Der Schwerpunkt eines Wellenpakets kann sich ja durchaus langsamer bewegen als jede einzelne Welle im Paket.

    Wie stellen die Elementarteilchenphysiker sich das vor?

  57. #57 MartinB
    7. Mai 2015

    @Gebhard
    Kräfte übertragen können nur Bosonen, also Teilchen mit ganzzahligem Spin. Stark eingedampft ist die Erklärung dafür, dass beim Austausch eines Fermions zwischen zwei Teilchen sich der Zustand der beiden teilchen, die das Fermion aussenden bzw. einfangen, unzulässig ändern würde, so dass man nie eine echte Kraft herausbekommen kann. (Das ist sehr vage erklärt, explizit vorgeführt wird das in Feynmans Lectures on Gravitation).

    Das mit der Geschwindigkeit hängt ein bisschen davon ab, wie man die Sache betrachtet – wenn man das Elektronfeld als Feld eines Teilchens mit Masse betrachtet, dann breitet es sich tatsächlich deutlich anders aus als z.B. eine Lichtwelle (eine Animation für Teilchen ohne Spin findet sich in Teil 5 oder 6 der QFT-Serie). Alternativ kann man auch das Higgsfeld mit einbeziehen – dann würde sich das Elektronfeld ohne Higgsfeld tatsächlich lichtschnell ausbreiten, aber die WeWi verhindert das.

    “Wie stellen die Elementarteilchenphysiker sich das vor?”
    Je mehr ich Bücher lese, desto mehr gewinneich den Eidnruck, dass die meisten Physikerinnen sich so was gar nicht vorstellen, sondern vor allem rumrechnen. (Wellenpakete kommen in den meisten QFT-Büchern gar nicht vor, rühmliche Ausnahme ist das Buch von Bob Klauber.)

  58. #58 JoJo
    11. Mai 2015

    @MartinM:

    Eine Frage zu virtuellen Photonen und der Kraftvermittlung zwischen 2 Elektronen:

    Irgendwo (glaub sogar in einem Deiner Beiträge) fand ich mal die Erklärung über den Impuls: Das virtuelle Photon bewirkt, dass sich die Wellenfunktionen der beteiligten Elektronen ändern. Dadurch ändern sich z.B.die Impulse entsprechend der Anziehung / Abstoßung — zumindest im statistischen Mittel.

    Was ist nicht verstehe: Werden die Impulse der Elektronen vor und nach dem Austausch des virtuellen Photons gemessen, dann ist die Impulsänderung nicht nach oben beschränkt. Oder sind die Messungen korreliert bzw. die Elektronen nach dem Austausch verschränkt, so dass bei Messung der Impulse die Impulserhaltung gewährleistet ist?

  59. #59 MartinB
    12. Mai 2015

    @JoJo
    Wenn man einzelne Feynman-Diagramme anguckt, bei denen die ein- und auslaufenden Elektronen jeweils scharf definierte Impulse haben, dann ist die Impulserhaltung dadurch gewährleistet, dass das ausgetauschte Photon ja genau den Austausch-Impuls überträgt.
    Wenn ich die Elektronen nachher in einem Überlagerungszustand habe (also keinen eindeutigen Impuls), dann muss es tatsächlich so sein, dass die beiden Zustände passend verschränkt sind, so dass die Impulserhaltung gewährleistet ist. Muss in meinen Augen so sein, weil ich jeden Zustand in seine Impulszustände zerlegen kann, für die dann das Argument oben gilt. (Interessanterweise habe ich das *nie* in irgendeinem Buch gelesen, obwohl es eine naheliegende Frage ist – ich bin mir aber trotzdem ziemlich sicher, dass es so ist.)

  60. #60 HansK
    Basel, Schweiz
    27. Februar 2016

    Wenn ich es richtig verstanden habe, beschleunigt ein Photon bei seiner Entstehung (z.B. in einer Kerze) “aus dem Stand” auf Lichtgeschwindigkeit.
    Ist dieser Mechanismus schon geklärt, so dass ich ihn als Normalo ohne Physik-/Mathe-Kenntnisse verstehen könnte?

  61. #61 MartinB
    27. Februar 2016

    @HansK
    Das ist einfach eine falsche Vorstellung – das Photon entsteht nicht erst und saust dann los. Kann man sich vielleicht ähnlich wie eine Wasserwelle vorstellen, die wird auch nicht erst erzeugt und bewegt sich dann, sondern bewegt sich sofort vom Ursprung weg.

  62. #62 Stefan
    München
    22. April 2016

    Frage: Haben virtuelle Photonen auch einen Drehimpuls von 1?

  63. #64 ggreiter
    München
    5. September 2016

    Vier FRAGEN zur Darstellung der Zustände eines Photons im Hilbertraum:

    Bisher habe ich es immer so verstanden, dass jeder Zustand eines Quants in einem dem Quant zugeordneten Hilbertraum einer Richtung entspricht, d.h. einer durch den Ursprung gehenden Geraden bestimmt durch einen Eigenvektor einer linearen Operation, welche Zustandsübergang bewirkt.

    Diese Operation entspreche der Observablen (also einer Eigenschaft des Quants).

    Völlig unklar ist mir:

    (1) Wie ergibt sich die Dimension des Hilbertraumes?

    (2) Nehmen wir nun an, das Quant sei ein Photon und die Observable sei “Polarisierung”. Müsste dann nicht jeder Polarisierungszustand durch einen 2-dimensionalen Teilraum des Gesamtraumes repräsentiert sein (als Polarisationsebene, die ja bekanntlich 2 Dimensionen hat)?

    (3) Und wie sähe dieser Teilraum aus für zirkuläre Polarisation?

    Nebenbei noch: Warum stellen nur Eigenvektoren, die einem reellen Eigenwert zugeordnet sind, einen Quantenzustand dar?

  64. #65 MartinB
    5. September 2016

    @ggreiter
    Das so richtig zu erklären, braucht wohl ein Kapitel in einem Physikbuch, ich schreibe hier mal schnell was hin, aber ich garantiere für nichts…
    (1) Aus der Zahl der möglichen Eigenzustände; letztlich durch Lösung der entsprechenden Gleichung, die die Situation beschreibt. (Also z.B. Kastenpotential mit unendlichen Wänden: Abzählbar unendlich viele Zustände; freies teilchen: überabzählbar unendlich viele Zustände)
    (2) Ja, es gibt zwei orthogonale Eigenzustände, also zweidimensionaler Raum. Siehe die Serie “Quantenmechanik verstehen”, da erkläre ich das im Detail.
    (3) Genauso, ist eine andere Basis desselben Hilbertraums.
    (4) Weil man Eigenwerte messen kann und physikalische Observable keine komplexen Messwerte haben.

  65. #66 PAS
    Schwelm
    29. September 2017

    Ausgehend von der ersten Beschreibung (a. Eine Anregung des em-Feldes mit Energie hν), darf ich davon ausgehen, dass sich ein Photon zugleich in alle Raumrichtungen ausbreiten kann, so wie elektromagnetische Wellen es tun?

  66. #67 MartinB
    30. September 2017

    @PAS
    Ja, im Prinzip schon. Je nachdem, wie es ausgesandt wird.
    Wenn z.B. ein einzelnes angeregtes Atom in den Grundzustand fällt und dabei ein Photon aussendet, dass haben wir erst einmal eine Art Photonen-Wellenfunktion, die sich im einfachsten Fall kugelförmig ausbreitet. Wenn ich dan das Photon irgendwo messe, kollabiert diese Wellenfunktion, das Photon wurde ja “hier” gemessen.

    Wenn ein Photon in einem Laser ausgesandt wird, ist seine Richtung dagegen vorgegeben, deswegen ist ein Laser ja gebündelt.

  67. #68 hans wm Körber
    Kiel
    30. April 2018

    Meine Forschungen haben mich zur Erkenntnis gebracht, was Photonen sind. Langer Rede kurzer Sinn: Gehen Sie auf meine Seite http://www.elektron.wiki und lesen Sie dazu in der Rubrik »Veroeffentlichungen« relevante Aufsätze, möglichst den letztdatierten.

  68. #69 MartinB
    30. April 2018

    @hans wm Körber
    Ich lasse den Link gern stehen, bitte aber, das hier nicht weiter zu diskutieren, auf diesem Blog sind absurde Privattheorien, die mit moderner Physik nichts zu tun haben, kein Thema.

  69. #70 hans wm Körber
    Kiel
    30. April 2018

    Vielen Dank für die schnelle Abschiebung in „absurde Privattheorien“! Nur in Hybris ist ein übereiltes Urteil ohne kritisches Nachlesen verständlich. Viele meiner Diskutanten kommen allerdings ins Grübeln, liefern Anregungen und tauschen Aspekte aus. Da Uni-Physik eine Glaubenslehre ist, zu der nicht nach einem WARUM gefragt werden darf, ist es nicht verwunderlich, daß Diskussion unerwünscht ist. Immer schön nachbeten, bitte!

  70. #71 MartinB
    30. April 2018

    @hanswmKörber
    Ich habe mir drei oder vier Seiten der “Veröffentlichungen” angeguckt, das hat gereicht.
    Weitere Diskussionen dazu sind wie gesagt unerwünscht, private Physiktheorien habe ich seit ich blogge einige Dutzend zu sehen bekommen, keine davon hielt auch nur einer einfachen Prüfung stand.

  71. #72 Jens Kluge
    Singapur
    23. November 2018

    “Die Zahl der Elektronen ist im abgeschlossenen System immer eindeutig bestimmt”: und wenn sich das System in einer Überlagerung von zwei Zuständen mit definierter Teilchenzahl befindet, was dann? Das ist doch das Gleiche wie bei der Überlagerung von zwei Energieeigenzuständen, da kann man doch auch nicht sagen wegen der Energieerhaltung muss das System unbedingt in einem Energieeigenzustand sein.

  72. #73 MartinB
    23. November 2018

    @Jens
    Aber für Elektronen gilt die Ladungserhaltung oder auch “Superauswahlregel” – eine Überlagerung aus Zuständen mit unterschiedlicher Gesamtzahl an Elektronen gibt es nicht. (Wenn man Positronen mitnimmt, kann man aber ne Überlagerung aus 1 El und 2 El+1Pos haben, weil dann die Ladung in beiden Zuständen gleich ist.)
    https://en.wikipedia.org/wiki/Superselection#Superselection_sectors

  73. #74 Jens Kluge
    Singapur
    23. November 2018

    Hmm, die Natur will es wohl so, obwohl es etwas unbefriedigend ist einfach zu sagen für verschiedene Drehimpuls- und Energiezustände darf man überlagern aber für verschiedene Ladungszustände nicht. Ohne Begründung lässt man sich nicht so gern etwas verbieten.

  74. #75 MartinB
    23. November 2018

    @Jens
    Letztlich muss die Physik halt die Natur beschreiben…
    Intuitiv denke ich, dass das mit der Quantisierung der Ladung zusammenhängt, aber belegen kann ich das nicht.

  75. #76 Stefan Berg
    Bremen
    4. Februar 2019

    wenn man Photonen verschränkt, ergibt sich ein Quantensystem. Beide Photonen reagieren dann instantan und simultan. Kann man diese spukhafte Fernwirkung nicht dadurch erklären, daß die Verbindung nicht abbricht, da ein einzelnes Photon immer mit dem “Lichtstrahl” verbunden bleibt?

  76. #77 MartinB
    4. Februar 2019

    @Stefan
    Ich weiß nicht, was du meinst – welcher “Lichtstrahl”? Ein Lichtstrahl besteht seinerseits aus Photonen. Zwischen zwei verschränkten Photonen ist kein verbindendr “Lichtstrahl”. Oder was genau meinst du?

  77. #78 Gebhard Greiter
    20. August 2019

    Wie erzeugt man eigentlich ein einzelnes Photon?

    Genauer: Ein Photon zu erzeugen muss man eine elektrische Ladung beschleunigt bewegen (was dann zu einer harmonischen Welle im elektromagnetischen Feld führt: dem Photon). Was aber gilt denn nun als Ende dieser Welle (= diesen Photons) und wie führt man es herbei?

  78. #79 MartinB
    20. August 2019

    @Gebhard
    Das vermischt so sehr klassische Bilder und Quantenmechanik, dass es keine sinnvolle Frage ist.

  79. #80 Gebhard Greiter
    20. August 2019

    Dann einfacher gefragt:

    Wie erzeugt man ein einzelnes Photon?
    Wie ich gelegentlich lese, soll das ja möglich sein.

  80. #81 MartinB
    20. August 2019

    Z.B. mit atomaren Übergängen, nem Photoeffekt (deswegen heißt der so) usw.

  81. #82 JörgR
    5. Februar 2020

    Hallo Herr Bäker,

    Sie hatten geschrieben:

    Bei einem Dipol habe ich tatsächlich eine Welle, die sich kugelförmig nach außen ausbreitet. Wenn ich mir die als “Paket” vorstelle (ich schalte den Dipol kurz an und sende ein pulsartiges Signal aus), dann bildet die Welle eine Kugelschale, die immer größer wird.
    Wenn ich mir jetzt vorstelle, in diesem Paket steckt nur die Energie eines einzigen Photons, dann breitet sich die Welle trotzdem erst mal kugelförmig aus. Ich habe jetzt überall eine Wahrscheinlichkeit, mein Photon zu messen. Anschaulich kannst du dir das auch als Überlagerung aus lauter einfachen Wellenpaketen vorstellen, die in alle Richtungen laufen.
    Wenn ich dann das Photon tatsächlich messe, dann wird eben dieser Zustand sozusagen “realisiert” (die “Wellenfunktion kollabiert”).

    Mit Ihrem Text wurde mir erstmals die Welle im Raum so erklärt, dass ich den Zusammenhang mit einem einzelnen Photon im Dipol-Szenario verstehe. Das war mir als altem Tecie mit Physikinteresse hilfreich.

    Statt vom Feld zum Photon zu kommen – umgekehrt:

    . Ein einzelnes kohärentes Photon startet nahe der Sonne und gelangt in mein Auge. Wie sieht im Raum die zugehörige (Summen-)Welle über die Flugzeit hinweg aus?
    . Wie verhält sich die Ausbreitungsrichtung der Welle zur Flugrichtung des Photons?

    In dieser thematischen Ecke fehlt mir auch die Vorstellung von Folgendem:

    . Es wird oft von der Ausbreitungsrichtung der Welle gesprochen – in einem linearen Sinn. Muss man sich das nicht (immer) im Raum vorstellen?
    . Wie sieht der Graph an der vordersten Front der Welle aus? Im linearen Fall?
    . Wie sieht der Graph der räumlichen Welle an einer Schnittebene aus? Genau wie eine aufrechterhaltene Wasserwelle mit punktuellem Anreger?
    . Welchen Weg nimmt ein Gravitron durch das Sonnensystem?
    . So wie Sie den “Lichtpuls” beschreiben: Ist denn jede Emission (z.B. von Photon) die Erzeugung einer (oder weniger) Wellenperiode?

    Mit Dank
    joerg

  82. #83 MartinB
    6. Februar 2020

    @JörgR
    Bin mir nicht ganz sicher, was genau du wissen willst. Man kann sich (vereinfacht) Ein einzelnes Photon als ein Wellenpaket vorstellen, das sich ausbreitet, so wie links in meinem header-Bild.

    Klar hat so ein Paket auch eine Querausdehnung, aber dafür habe ich keine saubere Anschauung (und keine Formeln) parat, insofern kann ich keine ganz genauen Skizzen machen wie so etwas aussehen würde. (Das Konzept einer “Photonenwellenfunktion” ist ohnehin etwas schwierig und knifflig.) Aber als Anschauung kann man sich schon ein Wellenpaket vorstellen, dass in Ausbreitungsrichtung läuft, und eigentlich sollte das reichen, um die meisten Phänomene prinzipiell zu beschreiben.

  83. #84 Jörg Rechner
    5. Mai 2020

    Hallo Herr Bäker,

    ich danke für Ihre eine Antwort. Jetzt hab ich meine Fragen unten neu formuliert.

    Mit Dank und erwartungsvoll
    joerg

    5 Experimente mit mehreren Fragen:

    1. Licht ist wohl eine elektromagnetische (em) Welle. Erst betrachte ich eine andere em Welle, von einem Dipol erzeugt – wie beim Radio. Nach dem Einschalten baut sich ein em Feld auf, das sich mit c quasi-rundum radial ausbreitet. Und in jedem Punkt des betroffenen Raumes schwingt dies Feld – in einer gewissen Richtung quer zur Ausbreitungsrichtung magnetisch und einem hierzu rechten Winkel elektrisch. Alles in der dipol-eigenen Frequenz. Ist das im Wesentlichen gesehen richtig?
    2. Jetzt betrachte ich eine Lichtquelle (in einem gewissen Spektrumbereich) statt des Dipols – also eine em-Welle anderer Frequenzen. Es ist wohl alles analog zu 1. – das quasi-rundum stellt sich wohl anders dar (vom breiten Strahlkegel bis zum punktuellen Laser variierend). OK?
    3. Wo sind in beiden Fällen die Photonen?
    Wann ist wo eines – bezogen auf die Punkte im Strahlungskegel und den Zeitverlauf?
    Auf jeder gedachten Kugel um die Quelle herum passieren wohl gleich viele Photonen – wenn keine absorbiert werden. Wieviele sind passierend pro Zeiteinheit – abhängig von der Strahlungsenergie und dem Kugelausschnitt?
    Wie oft wird ein Atom an einem Raumpunkt (gewissen Abstands von der Quelle) von einem Photon getroffen?
    Kann man das an einem praktischen/alltäglichen Beispiel quantifizieren?
    4. Ich gehe wohl recht, dass das Photon sich in Richtung der em-Wellenausbreitung bewegt (“fliegt”)? Man kann sich also
    – ein entstehendes und
    – ein fliegendes (d.h. geradlinig im Raum – und mit gleicher Geschwindigkeit, falls im Vakuum), wie auch
    – ein (momentan) absorbiertes
    Photon gut vorstellen. Und jedes (fliegendes) Photon ist ein Paket von Wahrscheinlichkeitswellen mit der Achse Flugrichtung und eben den Raum darum erfassend, d.h. in vielen Winkel beim Blick in Flugrichtung.
    Ist daran etwas falsch?
    5. Jetzt betrachte ich ein anderes Feld. Das Gravitationsfeld in unserem Sonnensystem – ein Feld das erst mal nicht schwingt. Aber durch die Wirkungen riesiger Bewegungen doch.
    Auf welcher Kurve fliegt hier ein Graviton von außerhalb startend durch dieses sonnenradiale Feld?
    Gibt es hier einen Wellencharakter und Pakete formal mit den gleichen Eigenschaften wie die des Photons?

  84. #85 MartinB
    5. Mai 2020

    @Joerg
    Zu 1: Ich denke schon.
    Zu 2: Ebenso.
    Zu 3: Verstehe die Frage nicht ganz.
    “Auf jeder gedachten Kugel um die Quelle herum passieren wohl gleich viele Photonen – wenn keine absorbiert werden. Wieviele sind passierend pro Zeiteinheit – abhängig von der Strahlungsenergie und dem Kugelausschnitt?”
    Ja, wenn die Lichtquelle die Photonen isotrop abstrahlt, dann ist das so,
    “Kann man das an einem praktischen/alltäglichen Beispiel quantifizieren?”
    Klar. Mit E=h nu kann man abschätzen, dass z.B. eine 11-Watt Energiesparlampe pro Sekunde etwa 2 mal 10 hoch 19 (2e19) Photonen aussendet, wenn du in 1m Entfernung stehst, treffen davon etwa 2e13 pro Sekude in dein Auge.

    Zu 4: “Paket von Wahrscheinlichkeitswellen” ist so halbwegs o.k., je nachdem, was du mit Wahrscheinlichkeitswelle meinst.

    Zu 5: Das Grav-Feld der Sonne ist analg nicht zu Licht, sondern zum E-Feld einer elektrischen Ladung. Entsprechend muss man da von virtuellen Teilchen mit entsprechender nicht-senkrechter Polarisation reden, aber das ist dann schon ziemlich kompliziert. In einem solchen Feld ist es auch nicht so sinnvoll, von einem “fliegenden” Graviton zu reden, das Feld ist ja statisch. Dafür verweise ich mal auf meine AFT-Serie oder auch auf mein Buch zur ART, wo ich das zumindest ein wenig erkläre.

    Bei Gravitationswellen ist es dann wieder anders, da passt die Analogie zu Licht halbwegs.

  85. #86 Tobias Wolfelsperger
    Burladingen
    22. Mai 2021

    Ist es nicht so das ein Photon alleine gar nicht bestehen kann. Das soll heissen das in jedem Photon wieder ein Photon enthalten ist usw. usw. Die Welt ist ein Fraktal. Eine immer wiederkehrende bis ins unendliche wiederspiegelung.
    Hierzu: http://www.fallwelt.de/NeuBerlin/physik2.pdf

    Des weiteren: https://www.cosmic-library.de/pdf/Physik%20und%20Metaphysik%20der%20Ummiten%20(2020).pdf

  86. #87 MartinB
    22. Mai 2021

    @Tobias
    Nein. Das ist pseudowissenschaftlicher Quatsch.

  87. #88 rolak
    22. Mai 2021

    fallwelt

    moin Tobias, vorab eine Frage zur Klärung: bist Du jetzt eine(r‍) der Aliens oder menschenstämmig erdgeboren?

    Folgende physikalische Theorien stammen aus einem Militärprojekt in dem Alienartefakte aus einer oder mehreren Alienbasen untersucht wurden.

  88. #89 Christian
    Wien
    6. Juli 2021

    Servus Martin!

    (1) Wie sieht unser Universum aus Sicht eines Photons aus? Einerseits vergeht für dieses aus unserer Perspektive keine Zeit. Da das Universum aus unserer Perspektive ein endliches Alter hat, befinden sich sämtliche Photonen im Zeitpunkt Null. Andereseits erscheint das Universum für das Photon in Bewegungsrichtung maximal Lorentz-verkürzt und entspricht somit einer unendlich dichten Fläche. Ist das Universum aus Sicht eines Photons tatsächlich nur zweidimensional und zeitlos?

    (2) Warum hat sich Einstein über verschränkte Photonen gewundert und von einer spukhaften Fernwirkung gesprochen? Aufgrund seiner Theorie hätte er doch wissen müssen, dass die Strecke zwischen zwei sich voneinander entfernenden Photonen maximal Lorentz-verkürzt und der Abstand dieser Photonen zueinander daher immer Null ist. Bei null Distanz instantan miteinander zu kommunizieren, ist doch etwas logisches und nichts komisches.

  89. #90 MartinB
    6. Juli 2021

    Moin,
    ein Photon hat/ist kein physikalisch sinnvolles Bezugssystem, deshalb sind solche Überlegungen wenig hilfreich. Da Photonen nichts messen oder wahrnehmen können, ist es ziemlich müßig, darüber zu spekulieren, wie sie die Welt wahrnehmen würden – wenn es die Sicht eines Photons gäbe, dann würde es aus seiner eigenen Sicht auch nur unendlich kurz existieren. Daraus ergeben sich keine physikalischen Folgerungen, also ist es auch nicht so schrecklich sinnvoll, darüber nachzudenken. (Dass es intuitiv trotzdem seltsam ist, ist natürlich richtig.)

    Zu 2: Die Fernwirkung findet ja messbar nicht für Photonen, sondern für uns in unseren Laboren statt. Wenn verschränkte Photonen echte Informationen austauschen würden, ergäbe sich damit ein Widerspruch zur Relativitätstheorie.

  90. #91 MartinB
    6. Juli 2021

    PS: Ich vergesse bei sowas immer, Werbung für mein Buch zu machen – ich erkläre das da aber auch einigermaßen detailliert.

  91. #92 Gebhard Greiter
    6. Juli 2021

    @MartinB #90:

    Die spannende Frage wäre, ob mit einander verschränkte Quanten nicht ohnehin als ein einziges Quantenobjekt aufgefasst werden müssen.

    Es ist ja schließlich der Zustand des einen nicht unabhängig vom Zustand des oder der anderen.

  92. #93 Stefan Freundt
    Kirchseeon
    6. Juli 2021

    Hallo Christian, die Frage ist sehr schön. Im Grund können wir nicht sagen, wie ein Photon aussieht. Am Ende sind alle Modelle unvollständig bzw. man muss sehr ingenieursmäßig zwischen einem Modell z.B. Welle und einem anderen Modell, Teilchen, hin und her springen. Letztlich haben wir keine Vorstellung, wie ein Photon, Elektron, Proton,… aussieht. Dann ist es natürlich unmöglich zu sagen, wie ein Photon die Welt sieht. Ich bin überzeugt davon, dass in diesem Fall schon das Bewusstsein für ein Problem sehr hilfreich ist.
    Hallo Martin,
    ihre Antworten sind immer sehr freundlich und sehr gut. Das schätze ich an Ihrem Blog. Eins nur:
    Photonen wechselwirken elektromagnetisch. Damit “sehen” (messen, wahrnehmen) sie Elektronen, Protonen, Quarks…
    VG
    Stefan

  93. #94 MartinB
    7. Juli 2021

    @Gebhard
    Sicher. Am Ende sind sie ja alle Anregungen eines einzigen Photonen-Felds.

    @Stefan
    Photonen wechselwirken nicht elektromagnetisch (das tun geladene Teilchen), sie sind die Träger der elektromagnetischen Wechselwirkung. Sie werden zwischen geladenen Teilchen ausgetauscht.
    Photonen selbst können nichts “messen”, weil sie bei solchen Prozessen zerstört bzw. erzeugt werden.

  94. #95 Stefan Freundt
    7. Juli 2021

    @Martin
    Bei der Photon-Streuung, zB. bei der Compton-Streuung, stößt das Photon mit einem Elektron und wird in Richtung, Impuls und Energie verändert. Es existiert vorher, hinterher und “sieht” in diesem Sinne das Elektron.
    Sie haben aber geschrieben, dass Photonen “nichts wahrnehmen können” …
    Photonen als Träger der elektromagnetischen Wechselwirkung sind virtuelle Photonen. Die sind hier aber hier nicht gemeint.(Mit reellen Photonen könnte man nur Abstoßung beschreiben…Und ja, virtuelle Photonen werden erzeugt und vernichtet um die elektromagnetische Wechselwirkung zu beschreiben.)
    Noch deutlicher wird alles beim Doppelspaltexperiment: Hier starten Photonen von einer Lichtquelle und “sehen” durchaus, ob sie durch einen oder zwei Spalte fliegen…

  95. #96 Christian
    Wien
    7. Juli 2021

    Danke an den Martin und Stefan für die bisherigen Antworten!
    Das war der Hintergrund meiner Frage “Wie sieht unser Universum aus Sicht eines Photons aus?”: Indem Theodor Kaluza zu Beginn der 1920-er-Jahre die ART um eine Raumrichtung erweiterte, konnte er Gravitation und Elektromagnetismus mit einem Gleichungssatz beschreiben. Entweder ist er mit einer falschen Annahme zu einem richtigen Ergebnis gekommen oder aber das Universum hat tatsächlich mehr als drei Raumrichtungen, von denen uns aber nur drei zugänglich sind. Schon Kaluza hatte keine Antwort darauf, wo denn die vierte Raumrichtung sein soll, während Oskar Klein diese Frage gut fünf Jahre später insofern löste, indem er annahm, dass diese Extradimension aufgerollt sein würde. Eine Annahme, die auch heute noch für die String-Theorien essenziell ist. Die Annahme des Aufgerolltseins ist also schon knapp 100 Jahre alt, aber bis jetzt hat sich kein experimenteller Nachweis dazu ergeben, noch irgendeine Analogie dazu in der Natur gefunden.
    So, und jetzt komme ich wieder zurück zum Photon: Könnten wir die Gestalt eines Photons direkt beobachten, dann wäre es in Bewegungsrichtung maximal Lorentz-verkürzt, also unendlich dünn. Es würde uns also beispielsweise als ein zweidimensionales Plättchen erscheinen. Dass ein Photon auf drei Raumrichtungen bezogen ruhemasselos ist, wundert mich daher nicht, weil es andernfalls unendlich dicht sein müsste. Was wäre, wenn sich etwa die baryonische Materie in einem Raum mit vier Richtungen analog zu den obigen Photonen ebenfalls nur mit Lichtgeschwindigkeit bewegen könnte? Dann wäre die vierte Raumrichtung unendlich verkürzt, für uns, die wir eben aus baryonischer Materie bestehen, also nicht direkt beobachtbar. Die vierdimensionale Masse der baryonischen Materie wäre Null, da sie aus vierdimensionaler Sicht aus unendlich dünnen Hyperplättchen bestünde.
    Da diese Hyperplättchen aus ihrer eigenen dreidimensionalen Sicht sehr wohl eine Masse haben, gehe ich davon aus, dass sich Photonen selbst innerhalb ihrer Welt mit zwei Raumrichtungen auch eine Masse zuschreiben würden.
    Dass ein Photon “aus seiner eigenen Sicht auch nur unendlich kurz existiert”, wie du, Martin, schreibst, klingt für mich eher danach, dass ein Photon keiner Zeit unterliegt, sich also aus seiner Sicht in einer zeitlosen, zweidimensionalen Welt befindet. Eigentlich müsste das dann bedeuten, dass es in einer Welt mit vier Raumrichtungen zwei Zeitrichtungen gibt, oder?
    Sollte sich an meiner Skizzierung hier nichts physikalisch grundlegend falsches finden, dann wäre es wert, diese Idee weiter zu verfolgen, denn mir scheint die Sache mit den aufgerollten Dimensionen doch recht zurechtgebastelt zu sein, einfach weil bisher niemanden etwas besseres eingefallen ist.

  96. #97 Gebhard Greiter
    http://greiterweb.de/spw/Zu-Physik.htm
    7. Juli 2021

    @Christian #96:
    Jedes Photon breitet sich aus als Kugelwelle um Hindernisse herum gebeugt (hinterm Doppelspalt z.B. so stark gebeugt, dass es mit sich selbst interferiert).

    Zudem sollte man nie vergessen:
    Geschwindigkeit als solche gibt es nicht: Es gibt nur Relativgeschwindigkeit. Und so können Objekte, die sich durch den Raum bewegen, gleichzeitig zu unterschiedlichen Beobachtern unterschiedlich hohe Relativgeschwindigkeit haben. Es kann sich deswegen die Zeit auch nicht fürs reisende Objekt selbst verkürzen, sondern immer nur aus Sicht seiner Beobachter (und sie verkürzt sich auch keineswegs aus Sicht aller Beobachter im selben Ausmaß).

  97. #98 ggreiter
    greiterweb.de/spw/Zu-Physik.htm
    7. Juli 2021

    Zu #96 und #97:
    Man könnte es auc so ausdrücken:

    Zeit als solche gibt es nicht. Es gibt stattdessen nur beobachtete Zeiten. Zeit, die ein Beobachter an sich selbst beobachtet, nennt man seine Eigenzeit.

  98. #99 Stefan Freundt
    7. Juli 2021

    @Christian,
    ich bin kein Fan von extra-Dimensionen im Allgemeinen und der String-Theorie im Besonderen. Letztere kann das Doppelspalt-Experiment auch nicht erklären, ja hat dies nicht einmal als Ziel.
    Mit zunehmender Zeit habe ich auch Zweifel daran, dass drei Dimensionen im Bereich unterhalb von 1nm für Einzelobjekte eine gute Idee sind.
    Die prägnanteste Überlegung dazu sieht wie folgt aus:
    Ca. 1685 hat Isaac Newton die Differential-Gleichung erfunden, um die Position der Planten im Sonnensystem zu bestimmen. Planeten sind von der Erde aus betrachtet sehr klein, deshalb ist die Annahme von Punkten (Punkt-Masse-Modell) gerechtfertigt. Und sie machen bei ihrer Bewegung keine Sprünge. Deshalb ist auch die Annahme eines Kontinuum gerechtfertigt. Wir haben also eine Mathematik,
    die bestens dafür geeignet ist, die Position (und Geschwindigkeit) von ansonsten unveränderlichen Objekten zu bestimmen. Planeten haben nur ihre unveränderliche Masse als Eigenschaft.
    Und jetzt betrachte ich die Paarvernichtung.
    Da haben wir vorher Objekte (Elektron und Positron) mit:
    – Masse (Ruhmasse)
    – Ladung
    – Spin=1/2
    Und hinterher haben wir 2 oder 3 Photonen mit
    – keine Masse (keine Ruhmasse)
    – keine Ladung
    – Spin=1
    Obwohl sich alles ändert, verwenden wir immer noch die gleiche (Planeten-) Mathematik dafür.
    Und um das Übel perfekt zu machen: Die Position ist nur die Eigenschaft, die wir weder von Elektron noch vom Photon genau kennen…

    Meines Erachtens ist die ganze infinitesimale Denkweise den Problemen in der Mikrowelt nicht angemessen..
    Meine Frage ist also nicht, ob wir 3 oder mehr Dimensionen haben, sondern ab welchen Abständen es
    sinnvoll ist von 3 Dimensionen sprechen und damit zu rechnen…und wie eine Mathematik aussehen könnte/müsste, die geeignet ist, Objekte in einander umzuwandeln (siehe Paarvernichtung).

  99. #100 MartinB
    7. Juli 2021

    @Stefan #95
    Schau dir das Feynman-Diagramm zum Compton-Effekt an, dann siehst du, dass das Photon absorbiert und ein anderes emittiert wird:
    https://de.wikipedia.org/wiki/Compton-Effekt

    Der Unterschied zwischen virtuellen und realen Teilchen ist nicht so klar (und sinnvoll) wie es oft dargestellt wird, Feynman hat mal sinngemäß gesagt, alle Teilchen, die wir beobachten, sind zwangsläufig virtuell, und da ist viel Wahres dran:https://scienceblogs.de/hier-wohnen-drachen/2012/01/13/qft-fur-alle-was-sind-virtuelle-teilchen/

    Und auch beim Doppelspalt gibt es meiner Ansicht nach keine sinnvolle Möglichkeit, den Versuch aus “Sicht” eines Photons zu beschreiben.

    @Christian
    “Könnten wir die Gestalt eines Photons direkt beobachten”
    Das ergibt keinen Sinn. Ein Photon hat keine beobachtbare “Gestalt”, es ist eine Anregung des Quantenfelds. Klar, man kann versuchen, sich dem mit Bildern zu nähern (tue ich ja auch im Artikel), aber das sind anschauliche Umschreibungen abstrakter Konzepte, nicht mehr.

    “Eigentlich müsste das dann bedeuten, dass es in einer Welt mit vier Raumrichtungen zwei Zeitrichtungen gibt, oder?”
    Nein. Oder besser gesagt: Welche beobachtbare physikalische Vorhersage oder Idee soll sich aus solchen Überlegungen ableiten lassen? Es führt wirklich und wahrhaftig zu nichts, zu versuchen, die Welt aus “Sicht” eines Photons zu beschreiben.

    @Stefan#99
    Dinge wie die Paarvernichtung lassen sich mit der Quantefeldtheorie mit nahezu perfekter Genauigkeit vorhersagen und beschreiben, mit genau der Mathematik, die auf den Ideen Newtons beruht. Man kann zwar argumentieren, dass diese Berechnungen problematisch sind (Dinge wie Renormierung z.B. sind konzeptionell durchaus nicht unproblematisch) und dass auf fundamentaler Ebene solche Beschreibungen zusammenbrechen müssen – aber bisher hat halt niemand dafür eine passende Lösung gefundne.
    Uns so leid es mir tut: Als laie mit anschaulichen Vorstellungen hat man hier keine Chance.
    Siehe auch hier:
    https://scienceblogs.de/hier-wohnen-drachen/2015/07/28/physik-anders-als-sie-aussieht/

  100. #101 Stefan Freundt
    9. Juli 2021

    @Martin:
    In der QFT geht man von Feynman-Diagrammen mit virtuellen Teilchen aus,
    berechnet dann die dazu gehörigen Integrale und kommt zu sehr genauen Ergebnissen.
    Niemand bezweifelt das.
    Aber in Ihrer ganzen Denkweise und obigen Argumentation schließen Sie von
    der Richtigkeit der Ergebnisse auf die Richtigkeit der Annahmen.
    Das ist mathematisch nicht zulässig.
    Alle physikalischen Theorien sind Aneinanderreihungen von Implikationen (Wenn-Dann-Schlußfolgerungen).
    Es ist aber unzulässig bei einer Implikation von der Richtigkeit
    der Ergebnisse auf die Richtigkeit der Annahmen zu schließen.

    Ihre ganze Argumentation ist damit – aus Sicht eines Mathematikers – grundlegend unzulässig.

    Aus zwei Gründen bin ich mir aber auch ziemlich sicher und dessen bewusst,
    dass Sie Ihre Argumentation aufrecht erhalten werden:
    1. Sie haben sich bereits dafür entschieden mich als Laien hinzustellen und sich zum Experten zu erheben.
    Und das 1. Grundgesetz des Bloggers lautet nun mal, niemals einen Fehler zuzugeben.
    2. (noch wichtiger) Kein theoretischer Physiker ist in der Lage obigen Fehler zu erkennen. (*)

    Nun, ich vermute, Sie werden Ihren Denkweise aufrecht erhalten und verlieren.
    Was Sie verlieren?
    Wahrhaftigkeit und Autorität.

    (*) Das ist nicht ganz richtig. Der theoretische Physiker Robert Betts Laughlin, Nobelpreis 1998
    für den fraktionierten Quanten-Hall-Effekt, ist dazu in der Lage.
    Offenbar besitzt er eine adequate mathematische Ausbildung.
    Er ist die einzige mir bekannt Ausnahme.

    @Christian: Ihre Frage ist berechtigt.
    Man kann sie heute nicht beantworten, weil wir kein mechanistisches Modell für ein Photon haben.
    Niemand kann bis heute Objekte mit entsprechenden Eigenschaften konstruieren,
    die bei Wechselwirkung unsere Welt erzeugen und damit erklären und nicht nur beschreiben.
    Alle Theorien, die Sie kennen oder von denen Sie gehört haben (Quantenmechanik, Quantenelektrodynamik,
    Quantenchromodynamik,…) sind Vorschriften, nach denen entsprechend ausgebildete Ingenieure
    das Ergebnis eines Experiments vorhersagen können.
    Aber das war ja nicht Ihre Frage.

    Bzgl. einer zweiten Zeit-Dimension stimme ich Martin zu und möchte den berühmten Physiker Francis Crick zitieren:
    “The job of theorists, especially in biology, is to suggest new experiments. A good theory makes not only predictions,
    but surprising predictions that then turn out to be true.
    (If its predictions appear obvious to experimentalists, why would they need a theory?)”

  101. #102 MartinB
    10. Juli 2021

    @Stefan
    “Aber in Ihrer ganzen Denkweise und obigen Argumentation schließen Sie von
    der Richtigkeit der Ergebnisse auf die Richtigkeit der Annahmen.”
    Nein, eigentlich tue ich das nicht, das klappt in der Physik in dieser Absolutheit nie. Wir wissen ja nie, ob nicht morgen jemand eine bessere Theorie erfindet, die unsere als Grenzfall enthält, aber ganz andere Konzepte braucht, so wie die Newtonsche Theorie ein Grenzfall der Relativitätstheorie ist. Wäre z.B. die Stringtheorie gültig, dann wären alle Feynman-Diagramme nur Näherungen an das, was die Stringstatsächlich tun. Dann ist die QFT immer noch als effektive Theorie gültig, weil sie (im Rahmen der jeweiligen Genauigkeit) korrekte Vorhersagen macht, aber auch nicht mehr.
    Ein bisschen habe ich das in diesem Klassiker diskutiert:
    https://scienceblogs.de/hier-wohnen-drachen/2010/12/27/spezielle-relativitatstheorie/

    Aber die Dinge, die du in deinem Kommentar #99 für irgendwie rätselhaft erklärst (Paarvernichtung), die sind es nunmal in der QFT nicht, die lassen sich dort (nahezu) perfekt erklären.
    *Das* ist es, was ich kritisiere – die Aussage, man solle/könne nicht die gleiche “Planetenmathematik” (was immer das sein soll, ich nehme mal an, du meinst Differentialrechnung?) für die Beschreibung solcher Prozesse verwenden, wenn die erfolgreiche Rechnung der QFT doch genau zeigt, dass das geht (Und Vorhersagen mit einer Genauigkeit von 10 Nachkommastellen sind ja nicht zu verachten.)

    Und *deshalb* sage ich, dass man mit laienhaften Vorstellungen keine Chance hat – um die Physik zu verbessern oder neue Theorien aufzustellen, muss man die bestehenden genau kennen und genau wissen, was sie zu leisten vermögen und was nicht.

    “Und das 1. Grundgesetz des Bloggers lautet nun mal, niemals einen Fehler zuzugeben.”
    Wär mir neu – ich habe hier auf den Scienceblogs schon öfter gesagt, dass ich was falsch gemacht habe oder Artikel nachträglich korrigiert, andere hier tun das auch.

    Zu diesem hier:
    ” damit erklären und nicht nur beschreiben”
    In der Physik ist IMHO kein Unterschied zwischen “erklären” und “beschreiben”:
    https://scienceblogs.de/hier-wohnen-drachen/2010/08/31/kann-die-physik-die-welt-erklaren/

  102. #103 Gebhard Greiter
    17. Juli 2021

    Die mit Abstand beste Antwort auf die Frage “Was ist ein Photon? scheint mir gegeben durch ein neues Experiment, das beschrieben und seinem Ergebnis nach interpretiert wird im Video https://www.youtube.com/watch?v=SDtAh9IwG-I .

    Es scheint uns zu sagen:

    1. Ein Photon entsteht erst und nur an der Stelle, an der es stirbt.

    2. Ansonsten existiert nur ein nicht-quantifiziertes elektromagnetisches Feld.

    und:

    3. Gezielt nur ein einziges Photon zu erzeugen und zum Doppelspalt zu senden, scheint gar nicht möglich zu sein.

    4. Zum Doppelspalt gesandtes, ihn durchfließendes Licht ist immer das ihn umspülende Feld als Ganzes (Summe von QuBits ist es da nur rein rechnerisch wenn man sich die Schwingung des Feldes durch Fouriertransformation in harmonische, unteilbare Teilwellen zerlegt denkt).

  103. #104 MartinB
    17. Juli 2021

    @Gebhard
    Gibt es dazu was zu lesen? Ich mag keine Videos….

  104. #105 Jörg Rechner
    münchen
    17. Juli 2021

    Zu Gebhards 2.:
    Du meinst wohl “nicht-quantisiert”. Jedes Feld hat Quantitäten – unbedingt. Und: ein emFeld kann nicht quantisiert sein – seine Werte/Größen sind kontinuierlich in Zeit und in Raum.
    Oder?
    “Photon” ist ein Quant – unweigerlich quantisiert. Und aus unserer Sicht quantifiziert – ein Unikat mit seiner Welllenläge.

  105. #106 Gebhard Greiter
    17. Juli 2021

    @ #104:
    Auch ich mag Videos nicht wirklich. Das hier aber hat Qualität (und wird im zweiten Teil auch sehr präzise). Jeder Quantenphysiker sollte es sich bis zum Ende hin ansehen, um sich dann erst seine Meinung darüber zu bilden.

    Das Experiment und sein Ergebnis sprechen für sich. Ein Papier dazu kenne ich nicht.

  106. #107 MartinB
    17. Juli 2021

    @gebhard
    Wen es ein Experiment mit Ergebnis gibt, muss es ja auch ein paper geben…

  107. #108 Gebhard Greiter
    17. Juli 2021

    @Jörg Rechner, #105:
    Wenn der Experimentator und ich das Ergebnis des Experiments richtig deuten, gilt:

    1, Was von der Lichtkanone erzeugt wird, ist nur aus recht grober Sicht heraus genau ein Photon, ist tatsächlich aber das Photon eingebettet in eine Wolke zusätzlicher, weit weniger energiereicher virtueller Photonen. Sie alle tragen zum Feld bei – so etwa wie eine Tasse Wasser, die man in einen Kübel gießt, der schon ein klein wenig Wasser enthält. Dieses “ein klein wenig Wasser” entspricht der elektromagnetischen, allgegenwärtigen Feldstärke im Labor um die Versuchsanordnung herum.

    2. Was dann durch den Doppelspalt fließt, ist (um im Vergleich zu bleiben) der so deutlich größer gewordene Inhalt des Kübels.

    3. Was sich auf dem Bildschirm hinterm Doppelspalt als Punkt auf lichtempfindlichem Materiel zeigt, ist die Portion von Feldanregungsenergie, die dort abgegeben wird, aber doch nur grob der Feldanregung entspricht, die das Gerät des Experimentators erzeugt hat (als Summe eines Photons und vieler deutlich weniger energiereicher virtueller Photonen).

  108. #109 Gebhard Greiter
    17. Juli 2021

    @Martin B, #107:
    Das mag ja sein, aber ich kenne es nicht (da ich bisher nur das Video fand).

  109. #110 Stefan Freundt
    17. Juli 2021

    @Gebhard Greiter: Der Autor macht Interferenz-Experimente mit einem HeNe-Laser.
    Ein solcher hat eine Koherenzlänge von ca 300m. Hat man ein Interferometer bei dem die Arm-Längen-Differenz
    kleiner ist, wird man auch immer Interferenz-Ringe oder -Streifen beobachten. Das gilt auch, wenn man die Intensität
    in den single-Photon-Bereich absenkt. In dem Sinne bietet das Experiment nichts Neues. Es wird dazu kein Paper geben.

    @Martin: “Und Vorhersagen mit einer Genauigkeit von 10 Nachkommastellen sind ja nicht zu verachten.”
    Wenn Sie mit der Quantenfeld-Theorie die Welt beschreiben, stimme ich Ihnen zu. Aber wenn Sie sagen, dies sei eine Erklärung,
    widerspreche ich Ihnen. Nach dem Standardmodell der Elementarteilchen ist ein Elektron punktförmig und strukturlos –
    und die Quantenfeldtheorie verlangt, dass es virtuelle Photonen erzeugt. Wie soll das Elektron das machen?
    Erklärung gleich/ungleich Beschreibung
    Wenn man den Mars über lange Zeit (Jahre) von der Erde aus beobachtet, beobachtet man von Zeit zu Zeit eine Rückläufigkeit.
    https://www.starobserver.org/ap100613/
    Von der Erde aus betrachtet, muss man sich fragen, warum der Mars von Zeit zu Zeit rückläufig ist.
    Ptolemäus hat dies durch Epizyklen, kleine Kreise, beschrieben(!).
    Aber Kopernikus hat es als Scheinbewegung erklärt.
    Ich erwarte einfach, dass man all diese Verrücktheiten in der Quantenmechanik erklären kann bzw. Fragen einfach “zerfallen”.
    (Verrückheiten: Das Elektron im H-Atom ist überall und nirgends.
    “Ein Photon entsteht erst und nur an der Stelle, an der es stirbt.”… und viele mehr)
    Wie sagt doch der Typ in “Newsroom” so treffend: “Um ein Problem zu lösen muss man als erstes erkennen, dass es eins gibt.”
    Selten hat die Menschheit so viel Wissen angehäuft und gleichzeitig so widersprüchliche Erklärungen akzeptiert.
    Irgendwer wird in den nächsten Jahren, vielleicht Jahrzehnten große Teile der modernen Physik in Trümmer legen.
    @Gebhard: Eins noch zum Schluss:
    Man kann Interferenz-Ringe auch mit dem Licht einer Wolfram-Glühwendel-Lampe erzeugen.
    Die Kohärenzlänge beträgt nur wenige µm, aber es geht und die Arme müssen dann entsprechend genau gleich lang sein.
    Ich habe das während meiner Promotion in Laseroptik ein paar Mal gemacht…

  110. #111 MartinB
    18. Juli 2021

    @Stefan
    Wie gesagt, erklären ist in meinen augen in der Physik “mit einem Modell beschreiben”, und das kann die QM für diese Probleme.
    Kopernikus hatte übrigens auch Epizyklen, nur weniger, das war sein großer Vorteil. (Ohne Epizyklen geht es nur mit Ellipsen, aber die Idee hatte erst Kepler.)

  111. #112 Stefan Freundt
    20. Juli 2021

    “Kopernikus hatte übrigens auch Epizyklen”
    Diese Bemerkung finde ich maßlos enttäuschend.
    Sagen Sie Bescheid, wenn Sie sie gelöscht haben.
    Bis dahin viele Grüße
    Stefan

  112. #113 MartinB
    20. Juli 2021

    @Stefan
    Verstehe ich nicht. Kopernikus’ Modell hatte Epizykel, nur weniger. Ohne Epizykel geht es nur mit Ellpisen, das war aber Kepler und deutlich später.

  113. #114 Stefan Freundt
    23. August 2021

    @Martin: Stellen Sie sich vor, Sie wären 12 Jahre und würden zum “Nordpol” des Sonnensystems, also einige Lichtstunden oberhalb der Sonne fliegen und würden dann einige Jahre die Planeten beobachten. Was würden Sie sehen?

  114. #115 MartinB
    23. August 2021

    @Stefan
    Sorry, ich spiele keine komischen Ratespielchen, wenn Du ein Argument hast, trage es einfach vor.

  115. #116 Stefan Freundt
    23. Oktober 2021

    Martin,
    wenn man im Internet, zB hier
    https://de.wikipedia.org/wiki/Geozentrisches_Weltbild#Ptolem%C3%A4isches_Weltbild
    und hier
    https://de.wikipedia.org/wiki/Epizykeltheorie
    liest, findet man:
    Ptolemäus verwendete 80 Epizykel und Kopernikus 34.
    Und erst Kepler brauchte gar keine mehr.
    Das ist etwa das Niveau eines 10-Jährigen, der lesen, schreiben und rechnen kann.
    Und für einen 10-Jährigen ist diese Schlußfolgerung auch in Ordnung.

    Aber ein guter Physiker würde sofort sehen, dass
    – Kopernikus im Wesentlichen gar keine Epizykel braucht, um die Rückläufigkeit der Planeten zu erklären
    – die Rückläufigkeit der Planeten ist hier nur eine Scheinbewegung
    – Ptolemäus braucht für jeden Planeten mindestens einen Epizykel
    – Damit sind beide Theorien qualitativ vollkommen unterschiedlich,
    Kopernikus liegt fast richtig und Ptolemäus komplett falsch.
    – Wie wir heute wissen, bewegen sich die Planeten auf ziemlich guten Kreisen um die Sonne,
    Die Exzentrizität der Ellipsen beträgt nur 5% bis 20%.
    – Für die damalige Zeit war Kopenikus’ Vorstellung schwierig zu akzeptieren, weil
    – man bei der Bewegung der Erde um die Sonne keine Fahrtwind spürte
    – man bei den Fixsternen keine Parallaxe beobachtete
    (die damalige Auflösung war viel zu schlecht)
    – Tycho Brahe, der den Mars 16 Jahre lang beobachtete, schaffte es aus den letzten beiden Gründen nicht,
    die Sonne in die Mitte des Planetensystems zu stellen.
    – Sich über diese Einwände hinweg zu setzten, das schaffte erst Kepler.

    “Sorry, ich spiele keine komischen Ratespielchen”
    Was ich Ihnen vorgeschlagen habe, war ein Gedankenexperiment.
    Albert Einstein – vielleicht haben Sie seinen Namen schon einmal gehört – war meisterlich darin.
    Ich nehme an, es hat ihm bei der Ausarbeitung seiner Relativitäts-Theorien geholfen.
    Warum Sie dazu nicht im Stande sind, ist mir rätselhaft.
    Jeder 12-Jährig ist dazu in der Lage.

    Sie hätten dann gesehen, dass sich die Planeten auf ziemlich guten Kreisen um die Sonne bewegen.
    Stefan

  116. #117 MartinB
    24. Oktober 2021

    @Stefan
    Du warst es, der impliziert hat, Kopernikus hätte keine Epizykel gebraucht. Mit reinen Kreisen lässt sich die beobachtete Bewegung eben nich tbeschreiben, auch wenn die Planetenbahnen “fast” kreisförmig sind, war man eben schon zu Kopernikus’ Zeiten in der Lage, zu sehen, dass Kreisbahnen im heliozentrischen Weltbild nicht funktionieren.

    Ich empfehle mal dieses Blogartikel zum Thema.
    https://scienceblogs.de/hier-wohnen-drachen/2010/08/28/eine-kritische-analyse-der-heliozentrischen-kosmologie/

    Deinen passiv-aggressiven Tonfall kannst du dir im übrige auch gern sparen.

  117. #118 Christian
    Wien
    24. März 2022

    Allgemein ist immer wieder davon die Rede, dass Elementarteilchen wie das Elektron oder Photon Punktteilchen, also nulldimensional seien. Mit einer Ruhemasse größer Null wäre ein Elektron somit eine Singularität mit einem Schwarzschildradius von 1,35*10^-57 m. (Exakter wäre wohl unter Berücksichtigung des Elektronenspins stattdessen die Kerr-Newman-Metrik anzuwenden.) Gut, das liegt zwar um 16 Größenordnungen unterhalb der Planck-Länge, aber die Masse des Elektrons liegt auch schon um 23 Größenordnungen unterhalb der Planck-Masse und trotzdem lässt sich die Elektronenmasse messen. Da 1,35*10^-57 m zwar sehr klein, aber nicht unendlich klein ist, empfinde ich die Behauptung der Nulldimensionalität des Elektrons wie sämtlicher Elementarteilchen mit einer Ruhemasse größer Null als Widerspruch. Überhaupt: Ließe sich über die Größe von Elementarteilchen nicht erst dann mit einiger Sicherheit etwas sagen, wenn es eine Theorie der Quantengravitation gäbe?

    Außerdem wurde gemessen, dass das Elektron eine praktisch perfekte Kugelform hat: https://www.weltderphysik.de/gebiet/teilchen/nachrichten/2011/wie-rund-ist-das-elektron/
    Klarerweise ließe sich argumentieren, dass die Auswirkungen eines Elektrons auf dessen dreidimensionale Umgebung nur deshalb perfekt kugelförmig sind, weil das Teilchen punktförmig sei, aber dann sehe ich eine Inkonsistenz dazu, dass etwa dem Proton und Neutron doch ein Radius von 8 bis 9 mal 10^-16 m zugeschrieben wird. Proton und Neutron setzen sich aus drei, wiederum punktförmig angenommenen Quarks zusammen, die eben einen Abstand zueinander einhalten, der sich aus einem Kräftegleichgewicht ergibt. Hier wird also das “Kräftegleichgewichtshalo” von Elementarteilchen schon als Länge gezählt, sobald ein Elementarteilchen alleine ist, aber offenbar nicht mehr. Das halte ich für inkonsistent!

    Analoges gilt für das Photon: Zwar ist ein Photon eine Anregung des Quantenfelds, hat eben aber auch Teilchencharakter. Mit einer Ruhemasse Null könnte das Photon zwar punktförmig sein, aber dann sehe ich einen Widerspruch zu dem, was der Gruppe um Zhangbu Xu am RHIC gelungen ist und was auch im obigen CERN-Foto zu sehen ist, nämlich Photonen zur Kollision zu bringen und damit Elektron-Positron-Paare zu erzeugen. Denn die Wahrscheinlichkeit, dass nulldimensionale Teilchen in einer dreidimensionalen Umgebung miteinander kollidieren geht gegen Null, oder ist eigentlich Null, außer es handelt sich um unendlich viele solcher Teilchen, wodurch eventuell eine Chance auf Kollisionen bestünde: ∞⋅0=? Kurz gesagt, ich halte auch das Elementarteilchen Photon für nicht nulldimensional, sondern für zweidimensional, da es ja maximal Lorentz-verkürzt ist.

    Oder???

  118. #119 MartinB
    25. März 2022

    @Christian
    Da muss man wirklich sehr aufpassen. Ein Elektron ist punktförmig heißt nur, dass man ein Elektron durch Messung beliebig gut lokalisieren kann. Ein Elektron “ist” kein punktförmiges Teilchen, denn man muss es mit den Mitteln der Quantenmechanik (QM) bzw. Quantenfeldtheorie (QFT) beschreiben, wonach der Ort eines Elektrons unbestimmt ist. (Ich folge der üblichen Interpretation der QM, es gibt auch andere Modellvorstellungen, z.B. die Pilotwellen…)
    Um ein Elektron auf einen Punkt zu “konzentrieren”, braucht man extrem hohe Energien (für einen mathematisch idealen punkt unendlich hohe energie). Sieht man auch z.B. am Wasserstoffatom. Das hat einen Durchmesser von etwa einen Angström, der einfach dadurch kommt, dass eine stärkere Konzentration des Elektrons auf einen kleineren Bereich eine höhere Energie bräuchte. Im Wasserstoffatom verhält sich ein Elektron also zunächst mal wie ein ausgedehntes Gebilde, nicht wie ein punkt.

    Eine andere Möglichkeit, sich das vorzstellen, ist die Methode der Pfadintegrale (siehe z.B. meine QFT-Serie oder die Serie “Ein Teilchen fliegt von A nach B”): Dabei beschreibt man das, was ein Elektron tut, indem man alle Möglichkeiten anguckt, das ein punktförmiges Teilchen etwas tun kann (z.B. von A nach B fliegen) und dann summiert man über alle diese Pfade. Das Teilchen “ist” aber nicht punktförmig, sondern es ergibt sich quasi erst als Summe über alle Einzelmöglichkeiten.
    Das ist z.B. in der Stringtheorie anders, da beschreibt man alle Prozesse über Pfade, die ein eindimensionales Gebilde (eben der String) beschreibt.

    Bei Proton und Neutron ist es so, dass die ja nicht bloß aus drei Quark sbestehen, sondern aus einer Ansammlung von Quarks und Gluonen. Und diese Ansammlung hat eine Größe, die man wirklich messen kann und die wird auch mit zunehmender Energie (angesehen von der Lorentz-Kontraktion, die dazu führt, dass man Kollisionsprozesse mit einem “Pfannkuchenmodell” beschreiben muss) nicht kleiner.

    Insofern ist “punktförmig” ein etwas problematischer Begriff – letztlich ist es mal wieder so, dass das, was man im Alltag drunter versteht, nicht das ist, was die PhysikerInnen damit meinen.

    Auf jeden Fall eine gute Frage – wenn ich mal wieder Zeit zum Bloggen habe, schreibe ich vielleicht mal mehr dazu.

  119. #120 Christian
    Wien
    25. März 2022

    > wenn ich mal wieder Zeit zum Bloggen habe, schreibe ich vielleicht mal mehr dazu.

    Ja bitte – und danke für die ausführliche Antwort!

  120. #121 Christian
    Wien
    25. März 2022

    > Insofern ist “punktförmig” ein etwas problematischer Begriff – letztlich ist es mal wieder so, dass das, was man im Alltag drunter versteht, nicht das ist, was die PhysikerInnen damit meinen.

    Gilt das so auch für das Photon?

  121. #122 MartinB
    25. März 2022

    @Christian
    Für das Photon gilt das sogar eher noch mehr; ein Elektron kann man immerhin lokalisieren, ohne es zu absorbieren. Formal haben Photonen anders als Elektronen nicht mal einen wohldefinierten Ortsoperator, wenn ich das richtig erinnere (hab schon lange nichts mehr mit QFT zu tun gehabt…)

  122. #123 Jörg Rechner
    München
    23. Januar 2023

    Hallo Herr Bäker,
    heute komme ich mit einer Bitte zu einem Thema, das ganz neben Ihren Photonen liegt, die mich auch beschäftigt haben: der freie Fall der Erde zur Sonne, wenn keine Umkreisung und keine weiteren Einflüsse wären – auch wenn es irreal ist.

    Vielleicht können Sie mir Links oder Erklärungen nennen – zu

    die Differentialgleichung für diese Bewegung,
    deren Lösung,
    Dauer des Falles und Geschwindigkeit beim Zusammenstoß, und
    zwar in beiden Fällen – klassisch und relativistisch

    Es wäre mir eine Freude – ich fand im WWW keine klare Antwort.

    Mit Dank
    J.Rechner

  123. #124 Karl-Heinz
    Graz
    23. Januar 2023

    @Jörg Rechner

    Welche Anfangsgeschwindigkeit soll die Erde haben?

  124. #125 MartinB
    24. Januar 2023

    @Jörg Rechner
    Klassisch bekommt man die Bewegungsgleichung, wenn man die reduzierte Masse einführt, dann hat man eine Gleichung für ein Teilchen. Ich glaube, im Buch von Kuypers (Theoretische Mechanik) ist das vorgerechnet, Wiki schreibt auch viel dazu:
    https://de.wikipedia.org/wiki/Zweik%C3%B6rperproblem

    Relativistisch (im Rahmen der ART) ist das nicht so einfach, Wiki ist da ganz gut:
    https://en.wikipedia.org/wiki/Two-body_problem_in_general_relativity
    ansonsten steht sehr viel auch im Buch “Exploring Black Holes” von Taylor und Wheeler.

    Es gibt auch Online-Tools,, die so etwas rechnen können, habe aber gerade keinen Link parat.

  125. #126 Jörg Rechner
    m
    28. Januar 2023

    @Karl-Heinz: 0 radial und 0 tangential. Ambesten einen Kreis annehmen statt Ellipse
    @MartinB; Danke sehr – harte Kost.

  126. #127 Karl-Heinz
    Graz
    29. Januar 2023

    @Jörg Rechner

    Was ich jetzt schon ohne Rechnung schon sagen kann.
    • Die Erde wird im freien Fall auf die Sonne fallen.
    • Die Geschwindigkeit des Zusammenstoßes wird kleiner als die Fluchtgeschwindigkeit der Sonne sein.
    Fluchtgeschwindigkeit der Sonne = 617,6 km/s.
    Um den relativistischen Einfluss grob abzuschätzen berechne ich mit der Fluchtgeschwindigkeit der Sonne den Lorenzfaktor γ =1 / √(1- (v/c)^2) = 1,000 002.

    •Der relativistische Einfluss kann damit vernachlässigt werden.

    Benötigst du noch die klassische Rechnung?

  127. #128 Karl-Heinz
    Graz
    29. Januar 2023

    @Jörg Rechner

    Steht die Erde plötzlich still, dann besitzt sie noch immer eine potenzielle Energie. Diese potenzielle Energie wandelt sich beim freien Fall auf die Sonne, in Bewegungsenergie um.
    W = G・m・M・(1/Radius_Sonne -1/Abstand_SonneErde) =(1/2) ・ m・v^2
    v = √ (2・G・M・(1/Radius_Sonne -1/Abstand_SonneErde))

    • Solare Gravitationskonstante (G · M) = 1,327.124.400.41 · 10^20 m3/s2
    • Mittlere Entfernung Sonner Erde = 149,6 · 10^9m
    • Äquatorradius Sonne = 696.342 km = 696.342.000 m

    Damit ergibt sich eine Einschlagsgeschwindigkeit der Erde auf die Sonne mit 615,96 km/s

    Was noch fehlt ist:
    die Differentialgleichung für diese Bewegung, deren Lösung, Dauer des Falles

    Ps: Normalerweise kreist die Erde um die Sonne mit einer Geschwindigkeit von 29,78 Kilometer pro Sekunde.

  128. #129 Jörg Rechner
    m
    29. Januar 2023

    Schönen Dank!
    Schön dargestellt.
    Über die Energieumwandlung zu gehen, ist clever und erspart DGL-Kalkül. Irgendwo müsste es doch die DGL geben – die einfachste Bewegung im Raum, neben dem ungestörten Körper.
    Berechnung für die modelhafte Strecke zum Mittelpunkt der Sonne ist auch interessant.

  129. #130 Karl-Heinz
    Graz
    29. Januar 2023

    @Jörg Rechner

    Irgendwo müsste es doch die DGL geben – die einfachste Bewegung im Raum, neben dem ungestörten Körper.
    Berechnung für die modelhafte Strecke zum Mittelpunkt der Sonne ist auch interessant.

    Ja natürlich, bitte nicht erschrecken.

    • Die Sonne befindet sich bei y=0
    • Die Einschlagsstelle befindet sich bei y =’Radius der Sonne’ =R_S
    • Die Erde bedindet sich bei y =’Abstand (ErdeSonne)’ =R_SE
    • M … Masse der Sonne
    • Solare Gravitationskonstante (G · M) = 1,327.124.400.41 · 10^20 m3/s2

    Die DiffGleichung lautet: y”(t) = -G*M / y(t)^2

    Allgemeine Lösung der Diffgleichung

    c1 müsste sein: c1 = – G * M / R_SE
    c2 musst selber bestimmen

    ——————————————————
    Zum Stöbern 🙂

    Zum Stöbern: inhomogenes Schwerefeld der Erde (freier Fall aus großer Höhe)

  130. #131 Karl-Heinz
    Graz
    29. Januar 2023

    @Jörg Rechner

    Um die Fallzeit zu berechnen nimm am besten die Formel ganz unten im Bild. Ich hoffe, dass sie stimmt.
    Formel für die Fallzeit

    H … Entfernung Sonne Erde = 149,6 * 10^9 m
    R … Radius der Sonne = 696.342 km = 696.342.000 m
    g … Fallbeschleunigung auf der Sonne = 274 m/s^2

  131. #132 Karl-Heinz
    Graz
    29. Januar 2023

    @Jörg Rechner

    Untere Grenze
    Die Fallzeit muss größer sein als
    H/v = 149,6 * 10^6 km/(615,96 km/s) =
    2,8 Tage sein.

  132. #133 Karl-Heinz
    Graz
    29. Januar 2023

    @Jörg Rechner

    So falls ich mich nicht verrechnet habe, schlägt die Erde in die Sonne in 64,5 Tagen ein und erreicht dabei eine Einschlagsgeschwindigkeit von 615,96 km/s

    Für was braucht du sowas? ;-).

  133. #134 Karl-Heinz
    Graz
    30. Januar 2023

    @Jörg Rechner

    Man kann auch sehr schön zeigen, dass man über : y”(t) = -G*M / y(t)^2 durch entsprechende Umformung auf die Energieumwandlung kommt.

  134. #135 Jörg Rechner
    München
    13. Februar 2023

    @Karl-Heinz
    Dank für die Mühe.
    Ich bin Mathematiker und Hobby-Naturbetrachter – auch bzw. Newton, .Gravitation, EM-Felder, Quanten. Durch dich sind meine Überlegungen bestätigt worden – nur bei v hatte ich mich verrechnet. Die Herleitung der DGL ist ein Kapitel für sich, d.h. fand ich nirgens.
    Alles Gute!
    joerg
    PS: schön dass uns MartinB dies ermöglicht.

  135. #136 Karl-Heinz
    Graz
    14. Februar 2023

    @Jörg Rechner

    Zu Beginn dachte ich, dass die Lösung der DGl eine gemähte Wiese sei. Was für ein Irrtum. 😉